Vous êtes sur la page 1sur 114

Measure and Integration:

Concepts, Examples and Exercises


INDER K. RANA
Indian Institute of Technology Bombay
India
Department of Mathematics, Indian Institute of Technology, Bombay, Powai, Mumbai 400076, India
Current address: Department of Mathematics, Indian Institute of Technology, Powai, Mumbai 400076, India
E-mail address: ikrana@iitb.ac.in

These notes were specially prepared for the participants of the first Annual
Foundation School, May 2004, held at IIT Bombay, India.
Abstract. These notes present a quick overview of the theory of Measure and Integration. For a more detailed and motivated text, the reader
may refer authors book:
An Introduction to Measure and Integration,
Narosa Publishers, Delhi, 1997
or,
An Introduction to Measure and Integration,Second Edition,
Graduate Text in Mathematics, Volume 45,
American Mathematical Society, 2002.

May, 2004 Mumbai 400076

Inder K. Rana

Contents

Chapter 1.

Classes of sets

1.1. Semi-algebra and algebra of sets

1.2. Sigma algebra and monotone class

Chapter 2.

Measure

2.1. Set functions

9
9

2.2. Countably additive set functions on intervals

14

2.3. Set functions on algebras

15

2.4. Uniqueness problem for measures

17

Chapter 3.

Construction of measures

19

3.1. Extension from semi-algebra to the generated algebra

19

3.2. Extension from algebra to the generated -algebra

20

3.3. Choosing nice sets: Measurable sets

22

3.4. Completion of a measure space

24

3.5. The Lebesgue measure

27

Chapter 4.

Integration

35

4.1. Integral of nonnegative simple measurable functions

35

4.2. Integral of nonnegative measurable functions

38

4.3. Intrinsic characterization of nonnegative measurable functions 42


4.4. Integrable functions

50

4.5. The Lebesgue integral and its relation with the Riemann
integral

55
vii

viii

Contents

4.6. L1 [a, b] as the completion of R[a, b]


Chapter 5.

Measure and integration on product spaces

59
63

5.1. Introduction

63

5.2. Product of measure spaces

65

5.3. Integration on product spaces: Fubinis theorems

69

5.4. Lebesgue measure on


Chapter 6.

R2

and its properties

Lp -spaces

75
79

6.1. Integration of complex-valued functions

79

6.2. Lp -spaces

82

6.3. L (X, S, )

86

6.4. L2 (X, S, )

87

6.5. L2 -convergence of Fourier series

93

Appendix A.

Extended real numbers

97

Appendix B.

Axiom of choice

101

Appendix C.

Continuum hypothesis

103

Appendix.

References

105

Appendix.

Index

107

Chapter 1

Classes of sets

1.1. Semi-algebra and algebra of sets

Concepts and examples:


1.1.1. Definition:
Let X be a nonempty set and let C be a collection of subsets of X. We say
C is a semi-algebra of subsets of X if it has the following properties:
(i) , X C.
(ii) A B C for every A, B C.
(iii) For every A C there exist n N and
Snsets C1 , C2 , . . . , Cn C such
c
that Ci Cj = for i 6= j and A = i=1 Ci .

1.1.2. Definition:
Let X be a nonempty set and F a collection of subsets of X. The collection
F is called an algebra of subsets of X if F has the following properties:
(i) , X F.
(ii) A B F, whenever A, B F.
(iii) Ac F, whenever A F.
1.1.3. Examples:
(i) Let X be any nonempty set. The collections {, X} and P(X) := {E | E
X} are trivial examples of algebras of subsets of X. The collection P(X)
is called the power set of X.
1

1. Classes of sets

(ii) The collection I of all intervals forms a semi-algebra of subsets of R. For


a, b R with a < b, consider the collection I of all intervals of the form
(a, b], (, b], (a, ), (, +). We call I the collection of all left-open,
right-closed intervals of R. It is easy to check that I is also a semi-algebra
of subsets of R.
(iii) The collection
(
F(I) :=

ER E=

n
[

Ik , Ik I, Ik I = for k 6= , n N .

k=1

is an algebra of subsets of R. So is the class


)
(
n
[
Ik I = for k 6= , n N .
:= E R | E =
Ik , Ik I,
F(I)
k=1

(iv) Let X be any nonempty set. Let


C := {E X | either E or E c is finite}.
Then C is an algebra of subsets of X.
(v) Let X and Y be two nonempty sets, and F and G semi-algebras of subsets
of X and Y , respectively. Let
FG = {F G | F F, G G}.
Then, FG is a semi-algebra of subsets of XY .

Exercises:

(1.1) Let F be any collection of subsets of a set X. Show that F is an


algebra if and only if the following hold:
(i) , X F.
(ii) Ac F whenever A F.
(iii) A B F whenever A, B F.
(1.2) Let F be an algebra of subsets of X. Show that
(i) If A, B F then AB := (A \ B) (B \ A) F.
(ii) If E1 , E2 , . . . , En F then F1 , F2 , . . . ,SFn F such
S that
Fi Ei for each i, Fi Fj = for i 6= j and ni=1 Ei = nj=1 Fj .

The next set of exercise describes some methods of constructing algebras


and semi-algebras.

1.1. Semi-algebra and algebra of sets

(1.3) Let X be a nonempty set. Let =


6 E X and let C be a semialgebra (algebra) of subsets of X. Let
C E := {A E | A C}.
Show that C E is a semi-algebra (algebra) of subsets of E. Note
that C E is the collection of those subsets of E which are elements
of C when E C.
(1.4) Let X, Y be two nonempty sets and f : X Y be any map. For
E Y, we write f 1 (E) := {x X | f (x) E}. Let C be any
semi-algebra (algebra) of subsets of Y. Show that
f 1 (C) := {f 1 (E) | E C}
is a semi-algebra (algebra) of subsets of X.
(1.5) Give examples of two nonempty sets X, Y and algebras F, G of
subsets of X and Y, respectively such that F G := {A B | A
F, B G} is not an algebra. (It will of course be a semi-algebra,
as shown in example 1.1.3(v).)
(1.6) Let {F
T }I be a family of algebras of subsets of a set X. Let
F := I F . Show that F is also an algebra of subsets of X.
(1.7) Let {Fn }n1 be a sequence of algebras of subsets of S
a set X. Under
what conditions on Fn can you conclude that F :=
n=1 Fn is also
an algebra?

(1.8) Let C be a semi-algebra of subsets of a set X. A set A X is called


a -set if there
S exist sets Ci C, i = 1, 2, . . . , such that Ci Cj =
for i 6= j and
i=1 Ci = A. Prove the following:
S
(i) For any finite number of sets C, C1 , C2 , . . . , Cn in C, C\( ni=1 Ci )
is a finite union of pairwise disjoint sets from C and hence is a
-set.
S
(ii) For any sequence {Cn }n1 of sets in C,
n=1 Cn is a -set.
(iii) A finite intersection and a countable union of -sets is a -set.
(1.9) Let C be any collection of subsets of a set X. Then there exists a
unique algebra F of subsets of X such that C F and if A is any
other algebra such that C A, then F A. This unique algebra
given is called the algebra generated by C and is denoted by
F(C).

(1.10) Show that he algebra


generated by I, the class of all intervals, is
S
{E R | E = nk=1 Ik , Ik I, Ik I = if 1 k 6= n}.
(1.11) Let C be any semi-algebra of subsets of a set X. ShowSthat F(C), the
algebra generated by C, is given by {E X | E = ni=1 Ci , Ci C
and Ci Cj = for i 6= j, n N}.

1. Classes of sets

Remark:
Exercise 1.11 gives a description of F(C), the algebra generated by
a semi-algebra C. In general, no description is possible for F(C)
when C is not a semi-algebra.
S
(1.12) Let X be any nonempty set and C = {{x} | x X} {, X}. Is C
a semi-algebra of subsets of X? What is the algebra generated by
C? Does your answer depend upon whether X is finite or not?
(1.13) Let Y be any nonempty set and let X be the set of all sequences
with elements from Y, i.e.,
X = {x = {xn }n1 | xn Y, n = 1, 2, . . .}.
For any positive integer k let A Y k , the k-fold Cartesian product
of Y with itself, and let i1 < i2 < < ik be positive integers. Let
C(i1 , i2 , . . . , ik ; A) := {x = (xn )n1 X | (xi1 , . . . , xik ) A}.
We call C(i1 , i2 , . . . , ik ; A) a k-dimensional cylinder set in X
with base A. Prove the following assertions:
(a) Every k-dimensional cylinder can be regarded as a n-dimensional
cylinder also for n k.
(b) Let
A = {E X | E is an n-dimensional cylinder set for some n}.
Then, A {, X} is an algebra of subsets of X.
(1.14) Let C be any collection of subsets of a set X and let E X. Let
C E := {C E | C C}.
Then the following hold:
(a)
C E F(C) E := {A E | A F(C)}.
Deduce that
F(C E) F(C) E.
(b) Let
A = {A X | A E F(C E)}.
Then, A is an algebra of subsets of X, C A and
A E = F(C E).
(c) Using (a) and (b), deduce that F(C) E = F(C E).

1.2. Sigma algebra and monotone class

1.2. Sigma algebra and monotone class


Concepts and examples:
1.2.1. Definition:
Let X be any nonempty set and let S be a class of subsets of X with the
following properties:
(i) and X S.
(ii) Ac S whenever A S.
S
(iii)
i=1 Ai S whenever Ai S, i = 1, 2, . . . .

Such a class S is called a sigma algebra (written as -algebra) of subsets


of X.
1.2.2. Examples:
(i) Let X be any set. Then {, X} and P(X) are obvious examples of algebras of subsets of X.
(ii) Let X be any uncountable set and let
S = {A X | A orAc is countable}.
Then S is a -algebra of subsets of X.
T
(iii) Let X be any set and let C be any class of subsets of X. Let S(C) := S,
where the intersection is taken over all -algebras S of subsets of X such
that S C (note that P(X) is one such -algebra). It is easy to see that
S(C) is also a -algebra of subsets of X and S(C) C. In fact, if S is any
-algebra of subsets of X such that S C, then clearly S S(C). Thus
S(C) is the smallest -algebra of subsets of X containing C, and is called
the -algebra generated by C. In general it is not possible to represent
an element of S(C) explicitly in terms of elements of C.
1.2.3. Definition:
Let X be a nonempty set and M be a class of subsets of X. We say M is a
monotone class if
S
(i)
n=1 An M, whenever An M and An An+1 for n = 1, 2, . . . ,
T
(ii) n=1 An M, whenever An M and An An+1 for n = 1, 2, . . . .
1.2.4. Examples:

(i) Clearly, every -algebra is also a monotone class.

1. Classes of sets

(ii) Let X be any uncountable set. Let M := {A X | A is countable}.


Then M is a monotone class but not a -algebra.
(iii) Let X be any nonempty set and let C be any collection of subsets of X.
Clearly P(X) isTa monotone class of subsets of X such that C P(X).
Let M(C) :=
M, where the intersection is over all those monotone
classes M of subsets of X such that C M. Clearly, M(C) is itself a
monotone class, and if M is any monotone class such that C M, then
M(C) M. Thus M(C) is the smallest monotone class of subsets of
X such that C M(C). The class M(C) is called the monotone class
generated by C.

Exercises:

(1.15) Let S be a -algebra of subsets of X and let Y X. Show that


S Y := {E Y | E S} is a -algebra of subsets of Y.
(1.16) Let f : X Y be a function and C a nonempty family of subsets
of Y . Let f 1 (C) := {f 1 (C) | C C}. Show that S(f 1 (C)) =
f 1 (S(C)).
(1.17) Let X be an uncountable set and C = {{x} | x X}. Identify the
-algebra generated by C.
(1.18) Let C be any class of subsets of a set X and let Y X. Let A(C)
be the algebra generated by C.
(i) Show that S(C) = S(A(C)).
(ii) Let C Y := {E Y | E C}. Show that S(C Y ) S(C) Y.
(iii) Let
S := {E (B Y c ) | E S(C Y ), B C}.
Show that S is a -algebra of subsets of X such that C S
and S Y = S(C Y ).
(iv) Using (i), (ii) and (iii), conclude that S(C Y ) = S(C) Y.
(1.19) Let C be a class of subsets of a set X such that C. Then E S(C)
iff sets C1 , C2 , . . . in C such that E S({C1 , C2 , . . .}).
Hint:
The technique used to prove of exercise 1.19 is very useful, and
is often used to prove various properties of -algebras under consideration, is as follows: The sets satisfying the required property
are collected together. One shows that this collection itself is a
-algebra and includes a subfamily of the original -algebra which

1.2. Sigma algebra and monotone class

generates it. The claim then follows by the definition of the generated -algebra. We call this the -algebra technique.
(1.20) Let X be any topological space. Let U denote the class of all open
subsets of X and C denote the class of the all closed subsets of X.
(i) Show that
S(U) = S(C).
This is called the -algebra of Borel subsets of X and is
denoted by BX .
(ii) Let X = R. Let I be the class of all intervals and I the class of
all left-open right-closed intervals. Show that I S(U), I
I S(I) and hence deduce that
S(I),
= BR .
S(I) = S(I)
(1.21) Prove the following statements:
(i) Let Ir denote the class of all open intervals of R with rational
endpoints. Show that S(Ir ) = BR .
(ii) Let Id denote the class of all subintervals of [0, 1] with dyadic
endpoints (i.e., points of the form m/2n for some integers m and
n). Show that S(Id ) = BR [0, 1].
(1.22) Let C be any class of subsets of X. Prove the following:
(i) If C is an algebra which is also a monotone class, show that C is
a -algebra.
(ii) C M(C) S(C).
(1.23) (-algebra monotone class theorem ) Let A be an algebra of
subsets of a set X. Then , S(A) = M(A).
Prove the above statement by proving the following:
(i) M(A) S(A).
(ii) Show that M(A) is closed under complements by proving that
for
B := {E X | E c M(A)},
A B, and B is a monotone class. Hence deuce that M(A)
B.
(iii) For F M(A), let
L(F ) := {A X | A F M(A)}.
Show that E L(F ) iff F L(E), L(F ) is a monotone class,
and A L(F ) whenever F A. Hence, M(A) L(F ), for
F A.

1. Classes of sets

(iv) Using (iii), deduce that M(A) L(E) for every E M(A),i.e.,
M(A) is closed under unions also. Now use exercise (1.22) to
deduce that S(A) M(A).

Chapter 2

Measure

2.1. Set functions


Concepts and examples:
2.1.1. Definition:
Let C be a class of subsets of a set X. A function : C [0, +] is called
a set function. Further,
(i) is said to be monotone if (A) (B) whenever A, B C and
A B.
(ii) is said to be finitely additive if
!
n
n
X
[
(Ai ).

Ai =
i=1

i=1

whenever
A1 , A2 , . . . , An C are such that Ai Aj = for i 6= j
S
and ni=1 Ai C.

(iii) is said to be countably additive if


!

[
X

An =
(An )
n=1

n=1

whenever A1 , A2 , . . . in C with Ai Aj = for i 6= j and

n=1 An

C.
9

10

2. Measure

(iv) is said to be countably subadditive if


(A)

(An ).

n=1

whenever A C, A =

n=1 An

with An C for every n.

(v) is called a measure on C if C with () = 0 and is countably


additive on C.
Here are some more examples of finitely/countably additive set functions:
2.1.2.Example:
Let X be any infinite set and let xn X, n = 1, 2, . . . . Let {pn }n1 be a
sequence of nonnegative real numbers. For any A X, define
X
(A) :=
pi .
{i|xi A}

It is easy to show that is a countably additive set function on the algebra


P(X). We say is a discrete measure
with mass
P
Ppi at xi . The measure
is finite (i.e., (X) < +) iff i=1 pi < +. If i=1 pi = 1, the measure
is called a discrete probability measure/distribution. Note that
({xi }) = pi i and ({x}) = 0 if x 6= xi . So, one can regard as a set
function defined on the subsets of the set Y := {xn : n 1}. Some of the
special cases when X = {0, 1, 2, . . .} are:
(a) Binomial distribution: Y := {0, 1, 2, . . . , n} and, for 0 < p < 1,
 
n k
p (1 p)nk , 0 k n.
pk =
k
(b) Poisson distribution: Y := {0, 1, 2, . . .} and
pk := k e /k!
for k = 0, 1, 2, . . . , where > 0.
(c) Uniform distribution: Y := {1, 2, . . . , n},
pk := 1/k k.
An important example of a set function on the collection of intervals in
R.
2.1.3 Example:
We denote the set of real numbers by R. Let R denote the set of extended
real numbers.
Let I denote the collection of all intervals of R. If an interval I I has
end points a and b we write it as I(a, b). By convention, the open interval

11

2.1. Set functions

(a, a) = a R. Let [0, +] := {x R |x 0} = [0, +) {+}.


Define the function : I [0, ] by

|b a| if a, b R,
(I(a, b)) :=
+
if either a = or b = + or both.
The function , as defined above, is called the length function and has
the following properties:
Property (1): () = 0.
Property (2): (I) (J) if I J.
This is called the monotonicity property of (or one says that is
monotone).
S
Property (3): Let I I be such that I = ni=1 Ji , where Ji Jj = for
i 6= j. Then
n
X
(Ji ).
(I) =
i=1

This property of is called the finite additivity of , or one says that


is finitely additive.
S
Property (4): Let I I be a finite interval such that I
i=1 Ii , where
Ii I. Then

X
(Ii ).
(I)
i=1

Property (5): Let I I be a finite interval such that I =


In I and In Im = for n 6= m. Then
(I) =

n=1 In ,

where

(In ).

n=1

Property (6): Let I I be any interval. Then

X
(I) =
(I [n, n + 1)).
n=

Property (7): Let I I be any interval such that I =


and In Im = for n =
6 m. Then
(I) =

n=1

(In ).

n=1 In ,

In I

12

2. Measure

This property of is called the countable additivity of , or one says


that is countably additive.
Property (8): Let I I and I
(I)

n=1 In ,

In I. Then

(In ).

n=1

This property of is called the countable subadditivity of , or one


says that is countably subadditive.
Property (9): (I) = (I + x), for every I I and x R, where
I + x := {y + x | y I}.
This property of the length function is called translation invariance, or
one says that is translation invariant.

Exercises:
(2.1) Let X be any countably infinite set and let
C = {{x} | x X}.
Show that the algebra generated by C is
F(C) := {A X | A or Ac is finite}.
Let : F(C) [0, ) be defined by

0 if A is finite,
(A) :=
1 if Ac is finite.
Show that is finitely additive but not countably additive. If X is
an uncountable set, show that is also countably additive.
(2.2) Let X = N, the set of natural numbers. For every finite set A X,
let #A denote the number of elements in A. Define for A X,
#{m : 1 m n, m A}
.
n
Show that n is countably additive for every n on P(X). In a sense,
n is the proportion of integers between 1 to n which are in A. Let
n (A) :=

C = {A X | lim n (A) exists}.


n

Show that C is closed under taking complements, finite disjoint


unions and proper differences. Is it an algebra?

13

2.1. Set functions

(2.3) Let : I (0, 1] [0, ] be defined by



b a if a 6= 0, 0 < a < b 1,
(a, b] :=
+ otherwise.

(Recall that I (0, 1] is the class of all left-open right-closed intervals in (0, 1].)
Show that is finitely additive. Is countably additive also?

(2.4) Let X be a nonempty set.


(a) Let : P(X) [0, ) be a finitely additive set function
such that (A) = 0 or 1 for every A P(X). Let
U = {A P(X) | (A) = 1}.
Show that U has the following properties:
(i) 6 U.
(ii) If A X and B A, then B U.
(iii) If A, B U, then A B U.
(iv) For every A P(X), either A U or Ac U.
(Any U P(X) satisfying (i) to (iv) is called an ultrafilter
in X.)
(b) Let U be any ultrafilter in X. Define : P(X) [0, ) by

1 if A U,
(A) :=
0 if A 6 U.
Show that is finitely additive.

(2.5) Let A be an algebra of subsets of a set X.


(i) Let 1 , 2 be measures on A, and let and be nonnegative
real numbers. Show that 1 + 2 is also a measure on A.
(ii) For any two measures 1 , 2 on A, we say
1 2 if 1 (E) 2 (E), E A.
Let {n }n1 be a sequence of measures on A such that
n n+1 , n 1.
Define E A,
(E) := lim n (E).
n

Show that is also a measure on A and E B,


(E) = sup {n (E) | n 1}.
(2.6) Let X be a compact topological space and A be the collection of
all those subsets of X which are both open and closed. Show that
A is an algebra of subsets of X. Further, every finitely additive set
function on A is also countably additive.

14

2. Measure

2.2. Countably additive set functions on


intervals
Concepts and examples:
We saw in the previous section that the length function is a countably additive set function on the class of all intervals. One can ask the question:
do there exist countably additive set functions on intervals, other than the
length function? The answer is given by the following:
2.2.1. Proposition:
Let F : R R be a monotonically increasing function. Let F : I
[0, ] be defined by
F (a, b ] := F (b) F (a),
F (, b ] :=
F (a, ) :=
F (, ) :=

lim [F (b) F (x)],

lim [F (x) F (a)],

lim [F (x) F (x)].

Further, F
Then, F is a well-defined finitely additive set function on I.
is countably additive if F is right continuous.
One calls F the set function induced by F .
The converse of proposition 2.2.1 is also true.
2.2.2. Proposition:
Let : I [0, ] be a finitely additive set function such that (a, b] < +
for every a, b R. Then there exists a monotonically increasing function
F : R R such that (a, b] = F (b) F (a) a, b R. If is also
countably additive, then F is right-continuous.
(Hint: Define F as follows:

(0, x] if x > 0,
0
if x = 0,
F (x) :=

(x, 0] if x < 0.)


2.2.3. Remarks:
(i) In case (R) < +, a more canonical choice for the required function F
in proposition 2.2.2 is given by F (x) := (, x], x R.

15

2.3. Set functions on algebras

(ii) Propositions 2.2.1 and 2.2.2 completely characterize the non-trivial countably additive set functions on intervals in terms of functions F : R R
which are monotonically increasing and right continuous. Such functions
are called distribution functions on R. The set function F induced by
the distribution function F is non-trivial in the sense that it assigns finite
non-zero values to bounded intervals.
Exercises:
(2.7) Let F (x) = [x], the integral part of x, x R. Describe the set
function F .
(2.8) R. Show that F1 := F + is also a distribution function and
F = F1 . Is the converse true?
(2.9) (i) Let C be a collection of subsets of a set X and : C [0, ]
be a set function. If is a measure on C, show that is finitely
additive. Is monotone? Countably subadditive?
(ii) If C be a S
semi-algebra, then is countably subadditive iff A
C with A
i=1 Ai , Ai C implies
(A)

(Ai ).

i=1

2.3. Set functions on algebras


Concepts and examples:
In this section, we give some general properties of a set function defined
on an algebra A of subsets of an arbitrary set X.
2.3.1. Theorem:
Let A be an algebra of subsets of a set X and let : A [0, ] be a set
function. Then the following hold:
(i) If is finitely additive and (B) < +, then (B A) = (B) (A)
for every A, B A with A B. In particular, () = 0 if is finitely
additive and (B) < + for some B A.
(ii) If is finitely additive, then is also monotone.
(iii) Let () = 0. Then is countably additive iff is both finitely additive
and countably subadditive.

16

2. Measure

Another characterization of countable additivity of set functions defined


on algebras is given in the next theorem.
2.3.2. Theorem:
Let A be an algebra of subsets of a set X and let : A [0, ] be such
that () = 0.
(a) If is countably additive thenSthe following hold:
(i) For any A A, if A =
n=1 An , where An A and An
An+1 n, then
(A) = lim (An ).
n

This is called the continuity


from below of at A.
T
(ii) For any A A, if A =
A
n=1 n , where An A with An
An+1 n and (An ) < + for some n, then
lim (An ) = (A).

This is called the continuity from above of at A.


Conversely,
(b) If is finitely additive and (i) holds, then is countably additive.
(c) If (X) < +, is finitely additive and (ii) holds, then is countably additive.

Exercises:
(2.10 ) (i) In the proofs of part (ii) and part (c) of theorem 2.3.2, where do
you think we used the hypothesis that (X) < +? Do you think
this condition is necessary?
(ii) Let A be an algebra of subsets of a set X and : A [0, ] be
a finitely additive set function such that (X) < +. Show that
the following statements are equivalent:
(a) lim (Ak ) = 0, whenever {Ak }k1 is a sequence in A with
k
T
Ak Ak+1 k, and
k=1 Ak = .
(b) is countably additive.

(2.11 ) Extend the claim of theorem 2.3.2 when A is only a semi-algebra


of subsets of X.
(Hint: Use exercise 1.8)
(2.12 ) Let A be a -algebra and : A [0, ] be a measure. For any
sequence {En }n1 in A, show that

17

2.4. Uniqueness problem for measures

(i) (lim inf n En ) lim inf n (En ).


(ii) (lim supn En ) lim supn (En ).
(Hint: For a sequence {En }n1 of subsets of a set X,
lim inf En :=
n

n=1 k=n

Ek lim sup En :=
n

Ek . )

n=1 k=n

2.4. Uniqueness problem for measures


Concepts and examples:
The problem that we want to analyze is the following: Let be a -finite
measure on an algebra A of subsets of X. Let 1 and 2 be two measures
on S(A), the -algebra generated by the algebra A, such that 1 (A) =
2 (A) A A. Is 1 = 2 ?
2.4.1. Definition:
Let C be a collection of subsets of X and let : C [0, ] be a set function.
We say is totally finite (or just finite) if (A) < + A C. The
set function is said to be sigma finite (written as -finite) if there exist
pairwise disjoint
S sets Xn C, n = 1, 2, . . . , such that (Xn ) < + for every
n and X = n=1 Xn .
2.4.2. Examples:

(i) The length function on the class of intervals is -finite.


the algebra generated by left-open right-closed intervals
(ii) Let A = A(I),
in R. For A A, let (A) = + if A 6= and () = 0. Then is a
measure on A and it is not -finite. Let R be chosen arbitrarily and
fixed. Let A denote the algebra of subsets of R generated by A and {}.
Define for A A ,
(A) :=

+ if A \ {} =
6 ,
0 if either A = or A = {}.

It is easy to check that is also a measure on A and is not -finite.


(iii) Let X denote the set of rationals in (0,1] and let A be as in (ii) above.
Show that the -algebra X S(A) = P(X) and that every nonempty set in
the algebra X A has an infinite number of points. For any E X S(A)
and c > 0, define c (E) = c times the number of points in E. Show that
c is a measure on X S(A) = S(X A) and is not -finite.

18

2. Measure

2.4.3. Proposition:
Let 1 and 2 be totally-finite measures on a -algebra S. Then the class
M = {E S | 1 (E) = 2 (E)} has the following properties:
(i) M is a monotone class.
(ii) If S = S(A), and 1 (A) = 2 (A) A A, then 1 (A) = 2 (A) A
S(A).

Exercises:
(2.13 ) Let A be an algebra of subsets of a set X. Let 1 and 2 be -finite
measures on a -algebra S(A) such that 1 (A) = 2 (A) A A.
Then, 1 (A) = 2 (A) A S(A).
(2.14) Show that a measure defined on an algebra A of subsets of a set
X is finite if and only if (X) < +.

Chapter 3

Construction of
measures

Concepts and examples:

3.1. Extension from semi-algebra to the


generated algebra
3.1.1. Definition:
Let Ci , i = 1, 2 be classes of subsets of a set X, with C1 C2 . Let function
1 : C1 [0, +] and 2 : C2 [0, +] be set functions. The set
function 2 is called an extension of 1 if 1 (E) = 2 (E) for every E 1 .
3.1.2. Examples:
In example 2.4.2(ii), each is an extension of the measure . Similarly, in
example 2.4.2(iii) each c is an extension of .
Above examples show that in general a measure on an algebra A can
have more than one extension to S(A), the -algebra generated by A.
Our next theorem describes a method of uniquely extending a measure
from a semi-algebra to the algebra generated by it.
3.1.3. Theorem:
Given a measure on a semi-algebra C, there exists a unique measure
on
F(C) such that
(E) = (E) for every E C.
The measure
is called the extension of .
19

20

3. Construction of measures

[Hint: For E F(C), with E =


C, define

Sn

i=1 Ei

(E) :=

for pairwise disjoint sets E1 , . . . , En

(Ei ).]

i=1

Exercises:
(3.1) Let C be a collection of subsets of a set X and : C [0, ] be a
set function. If is a measure on C, show that is finitely additive.
Is monotone? Countably subadditive?
(3.2) If C be a semi-algebra,
then is countably subadditive iff A C
S
with A i=1 Ai , Ai C implies
(A)

(Ai ).

i=1

(3.3) Using theorem 3.1.3, show that length function, which is initially
defined on the semi-algebra I of all intervals, can be uniquely extended to a set function on F(I), the algebra generated. It is
worth mentioning a result due to S.M. Ulam (1930) which, under
the assumption of the continuum hypothesis, implies that it is
not possible to extend the length function to all subsets of R.
Theorem (Ulam): Let be a measure defined on all subsets of
R such that ((n, n + 1]) < n Z and ({x}) = 0 for every
x R. Then (E) = 0 for every E R.
3.1.3. Remark:
Ulams theorem shows the impossibility of extending the length function
from intervals to all subsets of R, assuming the continuum hypothesis. We
shall see later that similar results can be proved if one assumes the axiom
of choice. Ulams result uses the property of that ({x}) = 0 x R,
and the fact that ([n, n + 1]) < + for every n Z. In the later results
we shall use the translation invariance property of the length function .

3.2. Extension from algebra to the generated


-algebra

Concepts and Examples:


Given an arbitrary measure on an algebra A of subsets of a set X, our
aim is to try to extend to a class of subsets of X which is larger than

3.2. Extension from algebra to the generated -algebra

21

A. Intuitively, sets A in A are those whose size (A) can be measured


accurately. The approximate size of any set E X is given by the outer
measure as defined next. Recall that, for any nonempty set A [0, +],
we write inf(A) := inf A [0, +) if A [0, +) 6= , and inf(A) := +
otherwise.
3.2.1. Definition:
Let A be an algebra of subsets of a set X and : A [0, ] be a measure
on A. For E X, define
)
(

[
X

Ai E .
(E) := inf
(Ai ) Ai A,
i=1

i=1

The set function is called the outer measure induced by .

3.2.2. Proposition (Properties of outer measure):


The set function : P(X) [0, ] has the following properties:
(i) () = 0 and (A) 0 A X.
(ii) is monotone, i.e.,
(A) (B) whenever A B X.
(iii) is countably subadditive, i.e.,
(A)

(Ai ) whenever A =

i=1

(iv) is an extension of , i.e.,

Ai .

i=1

(A) = (A) if A A.
3.2.3. Remarks:
(i) A set function defined on all subsets of a set X is called an outer
measure if has properties (i), (ii) and (iii) in proposition 3.2.2. The outer
measure induced by is characterized by the property that if is any
outer measure on X such that (A) = (A) A A, then (A) (A).
In other words, is the largest of all the outer measures which agree with
on A.
(ii) In the definition of (E) the infimum is taken over the all possible
countable coverings of E. To see that finite coverings will not suffice, consider E := Q (0, 1), the set of all rationals in (0, 1), and
Snlet I1 , I2 , . . . , In
be any finite collection
of
open
intervals
such
that
E

i=1 Ii . Then it is
Pn

easy to see that i=1 (Ii ) 1. This will imply (E) 1 if only finite
coverings are considered in the definition of , which contradicts the fact
that (E) = 0, E being a countable set.

22

3. Construction of measures

3.2.4. Example:
Let
A := {A R | Either A or Ac is countable}.
It is easy to see that A is a -algebra. For A A, let (A) = 0 if A is
countable and (A) = 1 if Ac is countable. Then, is a measure on A. Let
be the outer measure induced by on P(R). It follows from proposition
3.2.2 that is countably subadditive on P(R).
If A R is countable, then clearly A A, and hence (A) = (A) = 0.
Further, (A) = 1 iff A is uncountable. Since, R = (, 0] (0, ) and
(R) = 1 < 2 = (, 0] + (0, ).
This shows that need not be even finitely additive on all subsets.
Exercises:
(3.4) Show that (E), as in definition 3.2.1, is well-defined.
(3.5) The set function (E) can take the value + for some sets E.
(3.6) Show that
)
(

[
X

Ai E .
(E) = inf
(Ai ) Ai A, Ai Aj = for i 6= j and
i=1

i=1

(3.7) Let X be any nonempty set and let A be any algebra of subsets of
X. Let x0 X be fixed. For A A, define

0 if x0 6 A,
(A) :=
1 if x0 A.

Show that is countably additive. Let be the outer measure


induced by . Show that (A) is either 0 or 1 for every A X,
and (A) = 1 if x0 A. Can you conclude that (A) = 1 implies
x0 A? Show that this is possible if {x0 } A.

3.3. Choosing nice sets: Measurable sets

Concepts and examples:


In the previous section we defined the notion of , the outer measure induced by on all subsets of X. We saw that (A) = (A), A A, but
in general need not be even finitely additive on P(X). Let us try to identify some subclass S of P(X) such that restricted to S will be countable
additive. This is the class S which we call the class of nice subsets of X.

3.3. Choosing nice sets: Measurable sets

23

But the problem is how to pick these nice sets? This motivates our next
definition.
3.3.1. Definition:
A subset E X is said to be -measurable if for every Y X,
(Y ) = (Y E) + (Y E c ).

(3.1)

We denote by S the class of all -measurable subsets of X. Note that


E S iff E c S , due to the symmetry in equation (3.1).
Thus, a set E X is a nice set if we use it as a knife to cut any subset
Y of X into two parts, Y E and Y E c , so that their sizes (Y E) and
(Y E c ) add up to give the size (Y ) of Y. Thus a nice set is in a sense
a sharp knife.
3.3.2. Theorem:
Let E X. Show that the following statements are equivalent:
(i) E S .
(ii) For every Y X,
(Y ) (Y E) + (Y E c ).
(iii) For every Y X, with (Y ) < +,
(Y ) (Y E) + (Y E c ).
(iv) For every A A,
(A) (A E) + (A E c ).
We give an equivalent definition of measurable sets when (X) < +.
3.3.3. Theorem:
Let (X) < +. Then E X is -measurable iff
(X) = (E) + (E c )

Next, we check that S is indeed the required collection of nice sets.


3.3.4. Proposition:
The collection S has the following properties:
(i) A S .
(ii) S is an algebra of subsets of X, S(A) S , and restricted to
S is finitely additive.

24

3. Construction of measures

(iii) If An S , n = 1, 2, . . . , then
S is countably additive.

n=1 An

S and restricted to

(iv) Let N := {E X | (E) = 0}. Then N S .


This together with proposition 2.4.3 gives us the following:
3.3.5. Theorem:
Let be a measure on an algebra A of subsets of a set A. If is -finite, then
there exists a unique extension of to a measure on S(A), the -algebra
generated by A.
3.3.6. Remark:
Theorems 3.2.2.(iv) and 3.3.4 together give us a method of constructing an
extension of a measure defined on an algebra A to a class S S(A) A.
Exercises:
(3.8) Identify the collection of -measurable sets for as in example
3.2.4.
(3.9) Let X = [a, b] and let S be the -algebra of subsets of X generated
by all subintervals of [a, b]. Let , be finite measures on S such that
([a, c]) = ([a, c]), c [a, b]. Show that (E) = (E) E S.
as given in proposition
(3.10) Let F be the measure on the algebra A(I)
2.2.1. Let F itself denote the unique extension of F to LF , the
-algebra of F -measurable sets, as given by theorem 3.3.4. Show
that
(i) BR LF .
(ii) F ({x}) = F (x) lim F (y). Deduce that the function F is
yx

continuous at x iff F ({x}) = 0.


(iii) Let F be differentiable with bounded derivative. If A R is
a null set, then F (A) = 0.
The measure F is called the Lebesgue-Stieltjes measure induced by the distribution function F.

3.4. Completion of a measure space


Concepts and examples:
Theorem 3.3.4 showed that, given a -finite measure on an algebra A of
subsets of a set X, can be extended to a unique measure on the algebra S of -measurable subsets of X, and S S(A). In this section
we describe the relation between S and the sets in S(A).

3.4. Completion of a measure space

25

We first give an equivalent ways of describing (E) for any set E


X, being the outer
Smeasure induced by . Let A denote the collection
of sets of the form i=1 Ai , Ai A.
3.4.1. Proposition:
For every set E X,

(E) = inf { (A) | A A , E A}


= inf { (A) | A S(A), E A}
= inf { (A) | A S , E A}.

3.4.2. Proposition:
For every E X, there exists a set F S(A) such that E F, (E) =
(F ) and (F \ E) = 0.
The set F is called a measurable cover of E.
3.4.3. Corollary:
Let E X. Then there exists a set K E, K S(A), such that (A) = 0
for every set A E \ K.
The set K is called a measurable kernel of E.
3.4.4. Definition:
Let X be a nonempty set, S a -algebra of subsets of X and a measure on
S. The pair (X, S) is called a measurable space and the triple (X, S, ) is
called a measure space. Elements of S are normally called measurable
sets.
Till now what we have done is that, given a measure on an algebra A of
subsets of a set X, we have constructed the measure spaces (X, S(A), ),
(X, S , ) and exhibited the relations between them. The measure space
(X, S , ) has the property that if E X and (E) = 0, then E S . This
property is called the completeness of the measure space (X, S , ).
The measure space (X, S(A), ) need not be complete in general. However,
S is obtainable from S(A) and N := {E X | (E) = 0} by
S = S(A) N := {E N | E S(A), N N }.
One calls (X, S , ) the completion of (X, S(A), ). This construction can
be put in a general context as follows.
3.4.5. Definition:
Let (X, S, ) be a measure space and let N := {E X | E N for some

26

3. Construction of measures

N S with (N ) = 0}. One says (X, S, ) is complete if N S. Elements


of N are called the -null subsets of X.
The abstraction of the relation between the measure spaces (X, S(A), )
and (X, S , ) is described in the next theorem.
3.4.6. Theorem:
Let (X, S, ) be a measure space and let N be the class of -null sets (as in
definition 3.4.5). Let S N := {E N | E S, N N } and S N :=
{E N | E S, N N }. Then S N = S N is a -algebra of subsets of
X. Let (E N ) = (E), E S, N N . Then is a measure on S N
and (X, S N , ) is a complete measure space, called the completion of the
measure space (X, S, ). (The measure space (X, S N , ) is also denoted
by (X, S, ).
Finally we describe the relation between on P(X) and on A.
3.4.7. Proposition:
Let be a measure on an algebra A of subsets of a set X and let be the
induced outer measure. Let E S be such that (E) < + and let > 0
be arbitrary. Then there exists a set F A such that (E F ) < .
3.4.8. Note:
Whenever (X, S, ) is a finite measure space with (X) = 1, it is called a
probability space and the measure is called a probability. The reason
for this terminology is that the triple (X, S, ) plays a fundamental role
in the axiomatic theory of probability. It gives a mathematical model for
analyzing statistical experiments. The set X represents the set of all possible
outcomes of the experiment, the -algebra S represents the collection of
events of interest in that experiment, and for every E S, the nonnegative
number (E) is the probability that the event E occurs. For more details
see Kolmogorov [9] and Parthasarathy [10].
Exercises:
(3.11 ) Let E X, and let G1 , G2 be two measurable covers of E. Show
that (G1 G2 ) = 0.
(3.12) Let E1 E2 E3 . . . be subsets of X. Then
!

En = lim (En ).
n=1

(3.13 ) Let E X, and let K1 , K2 be two measurable kernels of E. Show


that (K1 K2 ) = 0.

27

3.5. The Lebesgue measure

(3.14 ) Let N := {E X | (E) = 0}. Show that N is closed under


countable unions and
S = S(A) N := {E N | E S(A), N N },
where S is the -algebra of -measurable sets. Further, A S
(A) = (E), if A = E N, with E S(A) and N N .

3.5. The Lebesgue measure


Concepts and examples:
We now apply the extension theory of measures, developed in previous sections, to the particular case when X = R, A = A(I), the algebra generated
by all intervals, and on A is the length function as described in section

3.1. The outer measure , induced by the length function , on all subsets of R is called the Lebesgue outer measure and can be described as
follows: for E R,
(
)

X
[

(E) := inf
(Ii ) Ii I i, Ii Ij = for i 6= j and E
Ii .
i=1

i=1

The -algebra of -measurable sets, as obtained in section 3.4, is called the


-algebra of Lebesgue measurable sets and is denoted by LR , or simply
by L. The -algebra S(I) = S(A) := BR , generated by all intervals, is called

the -algebra of Borel subsets of R. We denote the restriction of to L or


BR by itself. The measure space (R, L, ) is called the Lebesgue measure space and is called S
the Lebesgue measure. We note that since
on I is -finite (e.g., R = +
n= (n, n + 1]), the extension of to BR is
unique. It is natural to ask the question:
What is the relation between the classes BR , L and P(R)?
As a special case of theorem 3.4.6, we have L = BR N , where
N := {N R | N E BR , (E) = 0}.
Thus,
BR L P(R).
The question arises:
Is BR a proper subset of L?
That is, are there sets in L which are not Borel sets?. First of all, we note

that Cantors ternary set C N L. Further, E C, then (E) = 0


and hence E L. In other words, P(C) L. Thus, the cardinality of L

28

3. Construction of measures

is at least 2c (here c denotes the cardinality of the real line, also called the
cardinality of the continuum). Since L P(R), we get the cardinality
of L to be 2c. On the other hand, BR is the -algebra generated by all
open intervals of R with rational endpoints. One can show that the algebra BR of Borel subsets of R has cardinality c, that of the continuum.
Thus, there exist sets which are Lebesgue measurable but are not Borel sets.
The actual construction of such sets is not easy. One such class of sets is
called analytic sets. An analytic set is a set which can be represented as a
continuous image of a Borel set. For a detailed discussion on analytic sets,
see Srivastava [38], Parthasarathy [29].
Since L, the class of all Lebesgue measurable subsets of R, has 2c elements, i.e., same as that of P(R), the natural question arises:
Is L = P(R)?
We stated earlier that, if we assume the continuum hypothesis, it is not
possible to define a countably additive set function on P(R) such that
({x}) = 0 x R. In particular, if we assume the continuum hypothesis,
we cannot extend to all subsets of R. Hence L =
6 P(R). What can be said
if one does not assume the continuum hypothesis? To answer this question,
one can either try to construct a set E R such that E 6 L ,or, assuming
that such a set exists, try to see whether one can reach a contradiction.
G. Vitali (1905), F. Bernstein (1908), H. Rademacher (1916) and others
constructed such sets assuming the axiom of choice (see appendix B). The
example of Vitali used the translation invariance property of the Lebesgue
measure, and that of Bernstein used the regularity properties of the Lebesgue
measure. Rademacher proved that every set of positive outer Lebesgue
measure includes a Lebesgue nonmeasurable set. Even today, more and
more nonmeasurable sets with additional properties are being constructed.
For example, one can construct nonmeasurable subsets A of R such that

(A I) = (I) for every interval I R. Of course, all these constructions


are under the assumption of the axiom of choice. Lebesgue himself did
not accept such constructions. In 1970, R. Solovay [37] proved that if one
includes the statement all subsets of R are Lebesgue measurable as an
axiom in set theory, then it is consistent with the other axioms of set theory
if the axiom of choice is not assumed. Construction of a nonmeasurable
set(due to Vitali), assuming the axiom of choice, is given in exercise (3.26).
We recall that the -algebra BR includes all topologically nice subsets
of R, such as open sets, closed sets and compact sets. Also, for E BR , if
we transform E with respect to the group operation on R, e.g., for x R,
consider E + x := {y + x | y E}, then E + x BR . For this, note that the
map y 7 x + y is a homeomorphism of R onto R, and hence E + x B
for every open set E. We leave it for the reader to verify (using -algebra

29

3.5. The Lebesgue measure

techniques) that this is true for all sets E BR . The relation of on L


with on topologically nice subsets of R and the question as to whether
E + x L for E L, x R, i.e., do the group operations on R preserve the
class of Lebesgue measurable sets, will be analyzed in this section. We give

below some properties of which are also of interest.


3.5.1. Theorem:

Let E R and (E) < +. Then, given > 0, there exists a set F which
is a finite disjoint union of open intervals and is such that

(E F ) < .
We give next some more characterizations of Lebesgue measurable sets.
3.5.2. Theorem:
For any set E R the following statements are equivalent:
(i) E L, i.e., E is Lebesgue measurable.
(ii) For every > 0, there exists an open set G such that

E G and (G \ E) < .
(iii) For every > 0, there exists a closed set F such that

F E and (E \ F ) < .
(iv) There exists a G -set G such that

E G and (G \ E) = 0.
(v) There exists an F -set F such that

F E and (E \ F ) = 0.
[Hint: Prove the following implications:
(i)

(ii)

(iv)

(i)

(i)

(iii)

(v)

(i).]

and

3.5.3. Note:
Theorem 3.5.2 tells us the relation between L, the class of Lebesgue measurable sets, and the topologically nice sets, e.g., open sets and closed sets.
The property that for E L and > 0, there exists an open set G E
with (G \ E) < can be stated equivalently as:
(E) = inf{(U ) | U open, U E}.

30

3. Construction of measures

This is called the outer regularity of . Other examples of outer regular


measures on R (in fact any metric space) are given in the exercise 3.25.
Another topologically nice class of subsets of R is that of compact subsets
of R. It is natural to ask the question: does there exist a relation between
L and the class of compact subsets of R? Let K be any compact subset of
R. Since K is closed (and bounded), clearly K BR L and (K) < +.
It is natural to ask the question: can one obtain (E) for a set E BR , if
(E) < +, from the knowledge of (K), K compact in R? The answer is
given by the next proposition.
3.5.4. Proposition:
Let E L with 0 < (E) < + and let > 0 be given. Then there exists a
compact set K E such that (E \ K) < .
On the set R, we have the group structure given by the binary operation of
the addition of two real numbers. We analyze the behavior of on L under
the map y 7 y + x, y R and x R fixed. We saw that A + x is a
Lebesgue measurable set whenever A is Lebesgue measurable and x R. It
is natural to ask the question: for A L and x R, is (A + x) = (A)?
The answer is given by the following:
3.5.5. Theorem (Translation invariance property):
Let E L. Then E + x L for every x R, and (E + x) = (E).
We saw that Lebesgue measure is the unique extension of the length function
from the class I of intervals to BR , the -algebra of Borel subsets of R. This
gave us a measure on BR with the following properties:
(i) For every nonempty open set U, (U ) > 0.
(ii) For every compact set K, (K) < +.
(iii) For every E BR ,
(E) = inf{(U ) | U open, U E},
= sup{(C) | C E, C closed}.
If (E) < +, then we also have
(E) = sup{(K) | K E, K compact}.
(iv) For every E BR and x R, E + x BR and (E + x) = (E).
Thus the Lebesgue measure is a translation invariant -finite regular
measure on BR . The question arises: are there other -finite measures on
BR with these properties? Obviously, if c > 0 then c defined by (c)(E) :=
c(E), E BR , is also a -finite measure and is translation invariant. In
fact the following hold:

3.5. The Lebesgue measure

31

3.5.6. Theorem:
Let be a measure on BR such that
(i) (U ) > 0 for every nonempty open set U R.
(ii) (K) < + for every compact set K R.
(iii) (E + x) = (E), E BR and x R.
Then there exists a positive real number c such that (E) = c(E)
E BR .
3.5.7. Note:
In fact the above theorem has a far-reaching generalization to abstract topological groups. Let us recall that the set of real numbers R is a group under
the binary operation +, the addition of real numbers. Also, there is a topology on R which respects the group structure, i.e., the maps (t, s) 7 t + s
and t t from R R R and R R, respectively, are continuous
when R R is given the product topology. In an abstract setting, if G is a
set with a binary operation and a topology T such that (G, ) is a group
and the maps G G G, (g, h) 7 g.h and G G, g 7 g 1 are
continuous with respect to the product topology on G G, one calls G a
topological group. Given a topological group, let BG denote the -algebra
generated by open subsets of G, called the -algebra of Borel subsets of G.
The question arises: does there exist a -finite measure on G such that it
has the properties as given in theorem 3.5.5? A celebrated theorem due to A.
Haar states that such a measure exists and is unique up to a multiplicative
(positive) constant if G is locally-compact. Such a measure is called a (right)
Haar measure on G. Theorem 3.5.5 then states that for the topological
group R, the Lebesgue measure is a Haar measure. Consider the group
(R \ {0}, ), where R \ {0} = {t R|t 6= 0} and is the usual multiplication
of real numbers. Let R \ {0} be given the subspace topology from R. It is
easy to show that R \ {0} is a topological group and, E BR\{0} ,
Z
1
d(x)
(3.2)
(E) :=
|x|
E
is a Haar measure on R \ {0}.

3.5.8. Note:
In the previous sections we have seen how the general extension theory,
as developed earlier, can be applied to the particular situation when the
semi-algebra is that of intervals and the set function is the length function.
More generally, if we consider the semi-algebra I of left-open right-closed
intervals in R and consider F : R R as a monotonically increasing
right continuous function, then we can construct a countably additive set
as in example 1.1.2. Using theorem
function F on the semi-algebra I,

32

3. Construction of measures

3.4.6, we can construct a complete measure F on a -algebra of subsets


of R which includes BR . This measure F is called the Lebesgue-Stieltjes
measure induced by the function F. Note that F has the property that
F (a, b] < + a, b R, a < b. Conversely, given a measure on BR
such that (a, b] < + a, b R, a < b, we can restrict it to I and,
using proposition 2.2.2, define a monotonically increasing right continuous
function F : R R such that the unique Lebesgue-Stieltjes measure F
induced by F is nothing but (by the uniqueness of the extension). Thus
measures on BR which have the property that (a, b] < + a < b
can be looked upon as a Lebesgue-Stieltjes measure F for some F. We
point out that it is possible to find different F1 , F2 : R R such that
both are monotonically increasing and right continuous and F1 = F2 . If
is finite measure, i.e., (R) < +, then it is easy to see that F (x) :=
(, x], x R, is a monotonically increasing right continuous function
such that = F . This F is called the distribution function of . When
(R) = 1, is called a probability and its distribution function F, which is
monotonically increasing and is right continuous with lim [F (x) F (x)]
x

= F (R) = 1, is called a probability distribution function.


Exercises:
(3.15) Let I0 denote the collection of all open intervals of R. For E X,
show that
(
)

X
[

(E) = inf
(Ii ) Ii I0 i, for i 6= j and E
Ii .
i=1

i=1

(3.16) Let E R and let > 0 be arbitrary. Show that there exists an

open set U E such that (U ) (E)+. Can you also conclude


that (U \ E) ?
(3.17) For E R, let
diameter(E) := sup{|x y| | x, y E}.

Show that (E) diameter(E).

(3.18) Show that for E R, (E) = 0 if and only if for every > 0,
there exist a sequence {In }n1 of intervals such that E
n=1 and

(
\
E)
<
.
Such
sets
are
called
Lebesgue
null
sets.Prove
n=1
the following:
(i) Every singleton set {x}, x R, is a null set. Also every finite
set is a null set.
(ii) Any countably infinite set S = {x1 , x2 , x3 , . . .} is a null set.
(iii) Q, the set of rational numbers, is a null subset of R.

33

3.5. The Lebesgue measure

(iv) Every subset of a null set is also a null set. S


(iv) Let A1 , A2 , . . . , An , . . . be null sets. Then
n=1 An is a null
set.
(v) Let E [a, b] be any set which has only a finite number of
limit points. Can E be uncountable? Can you say E is a null
set?
(vi) Let E be a null subset of R and x R. What can you say about
the sets E + x := {y + x | y E} and xE := {xy | y E}?
(vii) Let I be an interval having at least two distinct points. Show
that I is not a null set.
(viii) If E contains an interval of positive length, show that it is not
a null set. Is the converse true, i.e., if E R is not a null set,
then does E contain an interval of positive length?
(ix) Show that Cantors ternary set is an uncountable null set.

(3.19) Let E [0, 1] be such that ([0, 1] \ E) = 0. Show that E is dense


in [0, 1]

(3.20) Let E R be such that (E) = 0. Show that E has empty interior.
(3.21) Let {En }n1 be any increasing sequence of subsets (not necessarily
measurable) of R. Then,
!

En = lim (En ).

n=1

(3.22) Let E R. Show that the following statements are equivalent:


(i) E L.

(ii) (I) = (E I) + (E c I) for every interval I.


(iii) E [n, n + 1) L for every n Z.

(iv) (E [n, n + 1)) + (E c [n, n + 1)) = 1 for every n Z.

(3.23) Let A L and x R. Using theorem 4.2.2, show that


(i) A + x L, where A + x := {y + x | y A}.
(ii) A L, where A := {y | y A}.
(3.24) Let (X, d) be any metric space and let be a measure on BX , the
-algebra generated by open subsets of X, called the -algebra of
Borel subsets of X. The measure is called outer regular if
E BX ,
(E) = inf{(U ) | U open, U E},
= sup{(C) | C closed, C E}.

(3.3)

(i) If (X) < +, show that is outer regular iff for every E
BX and > 0 given, there exist an open set U and a closed

34

3. Construction of measures

set C such that


U E C and (U C ) < .
(ii) For A X, let
d(x, A) := inf{d(x, y) | y A}.
Show that for every A X, x 7 d(x, A) is a uniformly continuous function.
(Hint: |d(x, A) d(y, A)| d(x, y) x, y.)
(iii) Let (X) < + and
S := {E BX | (3.3) holds for E}.
Show that S is a -algebra of subsets of X.
(iv) Let C be any
T closed set in X. Show that C S.
(Hint: C =
n=1 {x X : d(x, C) < 1/n}.)
(v) Show that is outer regular on BX .

(3.25) Let E BR . Show that E + x BR for every x R.


(3.26) Let E L and x R. Let
xE := {xy | y E} and E := {x | x E}.

Show that E, xE L for every x E. Compute (xE) and


(E) in terms of (E).
(3.27) Example (Vitali)(Existence of nonmeasurable sets:
Define a relation on [0,1] as follows: for x, y [0, 1], we say x is
related to y, written as x y, if x y is a rational. Prove the
following:
(i) Show that is an equivalence relation on [0, 1].
(ii) Let {E }I denote the set of equivalence classes of elements
of [0, 1]. Using the axiom of choice, choose exactly one element
x E for every I and construct the set E := {x | I}.
Let r1 , r2 , . . . , rn , . . . denote an enumeration of the rationals in
[1, 1]. Let
En := rn + E, n = 1, 2, . . . .
Show that En Em = for n 6= m and En [1, 2] for every n.
Deuce that

[
En [1, 2].
[0, 1]
n=1

(iii) Show that E is not Lebesgue measurable.

Chapter 4

Integration

Unless stated otherwise, we shall work on a fixed -finite measure space


(X, S, ).

4.1. Integral of nonnegative simple measurable


functions
Concepts and examples:
4.1.1. Definition:
Let s : X [0, ] be defined by
s(x) =

n
X

ai Ai (x),

x X,

i=1

where n is some positive integer; a1 , a2 , . . . , an are nonnegative


extended real
S
numbers; Ai S for every i; Ai Aj = for i 6= j; and ni=1 Ai = X. Such a
function
Pn on (X, S)
Pn s is called a nonnegative simple measurable function
and i=1 ai Ai (x) is called a representation of s. We say i=1 ai Ai is
the standard representation of s if a1 , a2 , . . . , an are all distinct.
We denote by L+
0 the class of all nonnegative simple measurable functions on (X, S).

Note that s L+
0 iff s takes only a finite number of distinct values, say
a1 , a2 , . . . , an , the value ai being taken on the P
set Ai S, i = 1, 2, . . . , n.
And in that case its standard representation is ni=1 ai Ai . Also note that
the class L+
0 depends only upon the set X and the -algebra S; the measure
plays no part in the definition of functions in L+
0.
35

36

4. Integration

4.1.2. Examples:
(i) Clearly, if s(x) c for some c [0, +], then s L+
0.
(ii) For A X, consider A : X [0, +], the indicator function of the
set A, i.e., A (x) = 1 if x A and A (x) = 0 if x 6 A. Then A L+
0 iff
A S, for A = aA + bA c with a = 1 and b = 0.
(iii) Let A, B S. Then s = A B L+
0 , since s = AB .
(iv) Let A, B S. If A B = , then clearly A + B = AB L+
0.
4.1.3. Definition:
R
Pn
For s L+
s(x)d(x),
0 with a representation s =
i=1 ai Ai , we define
the integral of s with respect to , by
Z
n
X
s(x)d(x) :=
ai (Ai ).
i=1

The integral s(x)d(x) is also denoted by


exercise (4.1)).

sd. It is well-defined (see

4.1.4. Proposition:
For s, s1 , s2 L+
0 and R with 0, the following hold:
R
(i) 0 sd +.

(ii) s L+
0 and

(iii) s1 + s2 L+
0 and
Z

(s)d =

(s1 + s2 )d =

sd.

s1 d +

s2 d.

(iv) For E S we have sE L+


0 , and the set function
Z
E 7 (E) := sE d
is a measure on S. Further, (E) = 0 whenever (E) = 0, E S.
R
Before we proceed further, we observe
that
s(x)d(x) is well-defined,(see
R
Exercise 4.1). The properties of sd, for s L+
0 , are given by the next
proposition.

4.1. Integral of nonnegative simple measurable functions

37

4.1.5. Proposition:
Let s L+
0 . Then the following hold:
(i) If {sn }n1 is any increasing sequence in L+
0 such that limn sn (x)
= s(x), x X, then
Z
Z
sd = lim
sn d.
n

(ii)

sd = sup

nR

s d | 0 s s, s L+
0 .

Exercises:
(4.1) Show that for s L+
0,
following: let
n
X

s=

s(x)d(x) is well-defined by proving the

ai Ai =

m
X

bj Bj ,

j=1

i=1

where {A1 , . . . , An } and {B1 , . . . , Bm } are partitions of X by elements of S, then


(ii)
s =

n
X
i=1

ai

m
X

Ai Bj =

j=1

m
X

bj

j=1

n
X

Ai Bj .

i=1

(ii)
n
X
i=1

(iii)

ai (Ai ) =

m
X

bj (Bj ).

j=1

s(x)d(x) P
is independent of the representation of the function s(x) = ni=1 ai Ai .

(4.2) Let A, B S. Express the functions |A B | and A +B AB


as indicator functions of sets in S and hence deduce that they
belong to L+
0.
(4.3) Let s1 , s2 L+
0 . Prove
R the following:
R
(i) If s1 s2 , then s1 d s2 d.
(ii) Let x X,
(s1 s2 )(x) := max{s1 (x), s2 (x)} and (s1 s2 )(x) := min{s1 (x), s2 (x)}.
Then s1 s2 and s1 s2 L+
0 with
Z
Z
Z
(s1 s2 )d si d
(s1 s2 )d, i = 1, 2.

38

4. Integration

(4.4) Express the functions A B and A B , for A, B S, in terms


of the functions A and B .
(4.5) Let X = (0, 1], S = B(0,1] , the -algebra of Borel subsets of (0, 1]
and = , the Lebesgue measure restricted to
PS. For x n (0, 1], if
x has non-terminating dyadic expansion x =
n=1 xn /2 . Let

+1 if xi = 1,
fi (x) :=
1 if xi = 0,
i = 1, 2, . . . .
+
Show that for every i, there
R exists simple function si L0 such
that fi = si 1. Compute si d.

(4.6) Let s : X R be any nonnegative function such that the range


1

of s is a finite set. Show that s L+


0 iff s {t} S for every t R .
(4.7) For s1 , s2 L+
0 show that {x | s1 (x) s2 (x)} S. Can you say
that the sets {x X | s1 (x) > s2 (x)}, {x X | s1 (x) s2 (x)} and
{x X | s1 (x) = s2 (x)} are also elements of S?
(4.8) Let s1 , s2 L+
0 be real valued and s1 s2 . Let = s1 s2 . Show
that L+
.
Can
you say that
0
Z
Z
Z
d = s1 d s2 d?

(4.9) Let {sn }n1 and {sn }n1 be sequences in L+


0 such that for each

x X, both {sn (x)}n1 and {sn (x)}n1 are increasing and

lim sn (x) = lim sn (x).

Show that
lim

sn d = lim

sn d.

(Hint: Apply exercise (4.3)Rand proposition 4.1.5


R to {sn sm }n for

all fixed m to deduce that sm d limn sn d.)

(4.10) Show that in general L+


0 need not be closed under limiting operations. For example, consider the Lebesgue measure space (R, L, )
and construct a sequence {sn }n1 in L+
0 such that lim sn (x) = f (x)
exists but f 6 L+
0.

4.2. Integral of nonnegative measurable


functions
Concepts and examples:
Having defined the integral for functions s L+
0 , i.e., nonnegative simple
measurable functions, we would like to extend it to a larger class.

4.2. Integral of nonnegative measurable functions

39

4.2.1. Definition:
(i) A nonnegative function f : X R is said to be S-measurable if
there exists an increasing sequence of functions {sn }n1 in L+
0 such that
f (x) = lim sn (x) x X.
n
If the underlying -algebra is clear from the context, a S-measurable
function is also called measurable. We denote the set of all nonnegative
measurable functions by L+ .
(ii) For a function f L+ , we define the integral of f with respect to by
Z
Z
sn (x)d(x).
f (x)d(x) := lim
n

R
It follows fromRexercise (4.9) that for f L+ , f d is well-defined.
Clearly,
R
+
+ and
L+

L
sd
for
an
element
s

L
is
the
same
as
sd,
for
s as an
0
0
+
element of L . The next proposition gives a characterization of functions in
L+ and the integrals of its elements. Another (intrinsic) characterization of
L+ will be given in the next section.
4.2.2. Proposition:
Let f : X R be a nonnegative function. Then the following hold:
(i) f L+ iff there exist functions sn L+
0 , n 1, such that 0 sn f n
and f (x) = lim sn (x) x X.
n
R
R
(ii) If f L+ and s L+
sd f d and
0 is such that 0 s f, then

Z
Z
+
f d = sup
sd 0 s f, s L0 .
4.2.3. Definition:
Let (X, S, ) be a measure space and Y S. We say a property P holds
almost everywhere on Y with respect to the measure if the set E = {x
Y | P does not hold at x} S and (E) = 0. We write this as P for a.e.
x() or P for a.e. ()x Y. If the set Y and are clear from the context,
we shall simply write P a.e. For example if f : X R is a function,
then f (x) = 0 for a.e. x() means that E = {x X | f (x) 6= 0} S and
(E) = 0.
R
We describe next the properties of f d, for f L+ .

4.2.4. Proposition:
Let f, f1 , f2 L+ . Then the following hold:
R
(i) f d 0 and for f1 f2
Z
Z
f1 d
f2 d.

40

4. Integration

(ii) For , 0 we have (f1 + f2 ) L+ and


Z
Z
Z
(f1 + f2 )d = f1 d + f2 d.

(iii) For every E S we have E f L+ . If


Z
(E) := E f d, E S,

then is a measure on S and (E) = 0 whenever (E) = 0.


R
R
The integral f E d is also denoted by E f d and is called the integral
of f over E.

(iv) If f1 (x) = f2 (x) for a.e. x(), then


Z
Z
f1 d = f2 d.

Since the class L+


0 is not closed under limiting operations (exercise 4.10),
we defined the class L+ by taking limits of sequences in L+
0 . Naturally, we
expect L+ to beRclosed under limits. The next theorem discusses this and
the behavior of f d under increasing limits, extending proposition 4.1.5
to functions in L+ .
4.2.5. Theorem (Monotone convergence):
Let {fn }n1 be an increasing sequence of functions in L+ , and f (x) :=
lim fn (x), x X. Then f L+ and
n
Z
Z
f d = lim
fn d.
n

4.2.6. Remark:
+
+
If {fn }n1
R is a sequence inR L decreasing to a function f L , then the
equality f d = limn fn d need not hold. For example, let X = R,
S = L and = , the Lebesgue measure. Let fn = [n,) . Then fn L+
0
R
+
n and
RL , and {fn }n1 decreases to f 0. Clearly, fn d = + for every
f d = 0. In fact, at this stage it is not clear whether f L+ whenever
{fn }n1 decreases to f , with each fn L+ . That this is true will be shown
as a consequence of the characterization of L+ proved in the next section.

41

4.2. Integral of nonnegative measurable functions

Exercises:
(4.11) Let f L+ and let {sn }n1 be in L+
0 and such that {sn (x)}n1
is decreasing and x X, lim sn (x) = f (x). Can you conclude
n
that
Z
Z
f d = lim

sn d?

(4.12) Let f L+ . Show that



Z
Z
.
f d = sup
sd 0 s(x) f (x) for a.e. x(), s L+
0

(4.13) Let {fn }n1 be an increasing sequence of functions in L+ such that


f (x) := lim fn (x) exists for a.e. x(). Show that f L+ and
n
Z
Z
fn d,
f d = lim
n

where f (x) is defined as an arbitrary constant for all those x for


which lim fn (x) does not converge.
n

(4.14) Let (X) < , and let f L+Sbe a bounded function. Let P :=
{E1 , E2 , . . . , En } be such that ni=1 Ei = X, Ei Ej = for i 6= j
and Ei S i. Such a P is called a measurable partition of X.
Given a measurable partition P = {E1 , . . . , En }, define
Mi := sup{f (x) | x Ei } and mi := inf{f (x) | x Ei }.
Let
P :=

n
X

mi Ei and P :=

n
X

Mi Ei .

i=1

i=1

Prove the following:


(i) For every partition P, show that P , P L+
0 and P f
RP .
R

(ii)
f d = supR P d | P is a measurable partition of X ,
= inf P d | P is a measurable
R partition of X .
(This gives an equivalent way of defining f d, in a way similar to that for the Riemann integral.)
(iii) Let
Z

+
= sup
sd | s L0 , s f
0

and

= inf

Z

sd | s

L+
0,


f s .

42

4. Integration

Show that

Z
p d | P is a measurable partition of X
= sup
and

= inf

Z

p d | P is a measurable partition of X .

(iv) Deduce that f L+ implies =

f d = .

Note:
R
Exercise 4.14 tells us that for f L+ , in defining f d it is enough
to consider approximations of f from below, Ras the approximations
from above will also give the same value for f d. This is because
f L+ , i.e., f is nonnegative measurable. The converse is also
true, i.e., if = , for f : X [0, ], then f L+ . To see this,
first note that
= < M ((X)) < ,
where M is such that |f (x)| M x X. Thus for every n we
can choose functions n , n L+
0 such that
Z
1
n f n and
(n n )d < .
n

Let := sup n and := inf n . Then , L+ (see corollary


4.3.11) and n,
Z
Z
1
( )d
(n n )d < .
n
Thus
Z
( )d = 0

and by proposition 4.3.3, = = f a.e. . Hence f L+ .

4.3. Intrinsic characterization of nonnegative


measurable functions
Concepts and examples:
Recall, f L+ means f : X R is a nonnegative function with the
property that there exists a sequence {sn }n1 of functions in L+
0 such that
{sn (x)}n1 increases to f (x) for every x X. Note that the measure
plays no part in the definition of the functions in L+ . It is only to define
the integral of functions f L+ that we need the measure . We want to
characterize the functions in L+ intrinsically.

4.3. Intrinsic characterization of nonnegative measurable functions

43

4.3.1. Proposition:
Let s : X R be such that s takes only finitely many distinct nonnegative
values. Then the following are equivalent:
(i) s L+ (and hence s L+
0 ).
(ii) s1 {t} S t R .
(iii) s1 [t, ] S t R .
(iv) s1 (I) S for every interval I in R .
In view of the above proposition it is natural to ask the following ques+
tion: does proposition 4.3.1 remain true if s L+
0 is replaced by any f L ?
The answer is yes, as proved in the next proposition.
4.3.2. Proposition:
Let f : X R be a nonnegative function. Then the following are equivalent:
(i) f L+ .
(ii) f 1 (c, +] S for every c R.
(iii) f 1 [c, +] S for every c R.
(iv) f 1 [, c) S for every c R.
(v) f 1 [, c] S for every c R.
(vi) f 1 {+}, f {} and f 1 (E) S for every E BR .
The statements (ii) to (vi) of proposition 4.3.2 describe the elements of
L+ intrinsically. This proposition is used very often to check the measurability of nonnegative functions. Once again we emphasize the fact that for
a nonnegative function f : X R to be measurable, i.e., for f to be in
L+ , the measure plays no part. As is clear from the above proposition, it
is only the -algebra S of subsets of X that is important . The notion of
measurability is similar to the concept of continuity for topological spaces.
As an immediate application of proposition 4.3.2, we have the following:
4.3.3. Proposition:
R
Let f L+ and E S be such that E f d = 0. Then f (x) = 0 for a.e.
()x E.
Next, we extend the concept of measurability of nonnegative functions
to functions f : X R which are not necessarily nonnegative.
4.3.4. Definition:
Let (X, S) be a measurable space.

44

4. Integration

(i) A function f : X R is said to be S-measurable if both f + and f


are S-measurable.
We denote by L the class of all S-measurable functions on X. If the
underlying -algebra is clear from the context, we call a S-measurable
function to be measurable.
(ii) If f L is such that both f + , f L+
0 , we call f a simple measurable
function.
We denote the class of all simple measurable functions by L0 . Note
that s L0 iff both s+ , s L+
0.
4.3.5. Proposition:
Let (X, S) be a measurable space and f : X R be any function. Then
following statements are equivalent:
(i) f is S-measurable.
(ii) There exists a sequence {sn }n1 of real valued functions on X such that

n, s+
n and sn are both nonnegative simple measurable functions on (X, S)
and lim sn (x) = f (x) x X.
n

(iii) f satisfies any one (and hence all) of the statements (ii) to (vi) of proposition 4.3.2.
4.3.6. Examples:
(i) Let f : X R be a constant function, f (x) = x X. Then f is
measurable c R, since


if c ,
{x X | f (x) > c} =
X if c > .

(ii) Let f : X R be measurable, and R. Then


since

{x X | f (x) < c/}

X
{x X | (x) > c} =

{x X | f (x) < c/}

f is also measurable,
if
if
if
if

> 0,
= 0, c 0,
= 0, c > 0,
< 0.

(iii) Let X be a topological space and S be the -algebra of Borel subsets


of X, i.e., the -algebra generated by the open sets. Let f : X R be
any continuous function. Then f is S-measurable.
We next describe some more properties of measurable functions.
4.3.7. Proposition:
Let f, g be measurable functions. Then each of the sets {x X | f (x) >

45

4.3. Intrinsic characterization of nonnegative measurable functions

g(x)}, {x X | {f (x) g(x)}, {x X | f (x) < g(x)}, {x X | f (x) g(x)}


and {x X | f (x) = g(x)} S.
4.3.8. Proposition:
Let f, g : X R be measurable functions and let R be arbitrary. Let
A := {x X | f (x) = +, g(X) = }{x X|f (x) = , g(x) = +}.
Define x X
(f + g)(x) :=

f (x) + g(x) if x 6 A,

if x A.

Then f + g : X R is a well-defined measurable function.


4.3.9. Proposition:
Let f : X R be measurable and let : R R be such that R {x
R | (x) } BR , R. Then f is also measurable.
4.3.10. Proposition:
Let fn : X R , n = 1, 2, . . ., be measurable functions. Then each of the
functions sup fn , inf fn , lim sup fn and lim inf fn is a measurable function.
n

In particular, if {fn }n1 converges to f , then f is a measurable function.


4.3.11. Corollary:
Let {fn }n1 be a sequence in L+ . Then each of the functions sup fn , inf fn ,
n

lim sup fn and lim inf fn is in L+ . In particular, if lim fn =: f exists, then

n
f L+ .

R
Recall that in theorem 4.2.5 we analyzed the limit of fn d for an increasing sequence
of nonnegative measurable functions. We analyze next the
R
behavior of fn d when {fn }n1 is not necessarily an increasing sequence
of nonnegative measurable functions.
4.3.12. Theorem (Fatous lemma):
Let {fn }n1 be a sequence of nonnegative measurable functions. Then
Z 
Z

lim inf fn d lim inf fn d.
n

4.3.13. Proposition:
Let (X, S) be a measurable space and let f : X R be S-measurable. Let
be a measure on (X, S). Let g : X R be such that {x X | f (x) 6=
g(x)} is a -null set. If (X, S, ) is a complete measure space, then g is also
S-measurable.

46

4. Integration

Exercises:
(4.15)(i) Let f L+ and E S beR such that f (x) > 0 for every x E
and (E) > 0. Show that E f d > 0.
(ii) Let f, g L+ be such that
Z
Z
Z
Z
f d = gd < + and
f d =
gd, E S.
E

Show that f (x) = g(x) a.e. (x).


(iii) Let f, g be nonnegative measurable functions on (R, L) such
that
Z b
Z b
f d =
gd < + for every a < b
a

show that then


Z

f d =

gd, E L,
E

and deduce that f (x) = g(x) a.e. (x).


(4.16) Let f : X R be a bounded measurable function. Then there
exists a sequence {sn }n1 of simple measurable functions such that
{sn }n1 converges uniformly to f.
(Hint: If 0 f (x) M x, then |sn (x) f (x)| < 1/2n n n0 ,
where n0 M and the sn s are as in proposition 4.3.2.)
(4.17) Let f : X R be a nonnegative measurable function. Show
that there exist sequences of nonnegative simple functions {sn }n1
and {
sn }n1 such that
0 sn (n) sn+1 (x) f (x) sn+1 (x) sn (x)
and lim sn (x) = f (x) = lim sn (x) x X.
n

(4.18) Let f and g : X


be measurable functions, p and R with
p > 1, and let m be any positive integer. Use proposition 4.3.9to
prove the following:
(i) f + is a measurable function.
(ii) Let and R be arbitrary. Define for x R,

(f (x))m if f (x) R,
m

if f (x) = +,
f (x) :=

if f (x) = .
Then f m is a measurable function.
(iii) Let |f |p be defined similarly to f m , where p is a nonnegative
real number. Then |f |p is a measurable function.

4.3. Intrinsic characterization of nonnegative measurable functions

47

(iv) Let , , R be arbitrary. Define for x R,

1/f (x) if f (x) 6 {0, +, },

if f (x) = 0,
(1/f )(x) :=

if
f (x) = ,

if f (x) = +.

Then 1/f is a measurable function.


(v) Let R be arbitrary and A be as in proposition 4.3.8.
Define for x R,

f (x)g(x) if x 6 A,
(f g)(x) :=

if x A.
Then f g is a measurable function.

(4.19) Let f : X R be S-measurable. Show that |f | is also Smeasurable. Give an example to show that the converse need not
be true.
(4.20) Let (X, S) be a measurable space such that for every function f :
X R, f is S-measurable iff |f | is S-measurable. Show that
S = P(X).
(4.21) Let fn L, n = 1, 2, . . . . Show that the sets
{x X | {fn (x)}n is convergent}
and
{x X | {fn (x)}n1 is Cauchy}
belong to S.
(4.22) Give an example to show that strict inequality can occur in Fatous
lemma.
P
(4.23) Let {fn }n1 be a sequence of functions in L+ and let
n=1 fn (x) =:
f (x), x X. Show that f L+ and
Z
Z
X
f d =
fn d.
n=1

(4.24) Show that each of the functions


f : R R defined below is LR
measurable, and
compute
f
d
:

0 if x 0,
(i) f (x) :=
1/x if x > 0.
(ii) f (x) := Q (x), the indicator function of Q, the set of rationals.

48

4. Integration

0 if x > 1 or x < 0 or x [0, 1] and x is rational,

n if x is an irrational, 0 < x < 1 and in the


(iii) f (x) :=
decimal expansion of x, the first nonzero

entry is at the (n + 1)th place.


R
R
(4.25) Let f , g L+ with f g. Show that (f g) L+ and f d gd.
Can you conclude that
Z
Z
Z
(f g)d = f d gd?
(4.26) Let f , fn L+ , n = 1, 2, . . ., be such that 0 fn f. If
lim fn (x) = f (x), can you deduce that
n

f d = lim

fn d?

(4.27) Let f L. For x X and n 1, define

f (x) if |f (x)| n,
n
if f (x) > n,
fn (x) :=

n if f (x) < n.

Prove the following:


(i) fn L and |fn (x)| n n and x X.
(ii) lim fn (x) = f (x) x X.
n

(iii) |fn (x)| := min{|fn (x)|, n} := (|f | n)(x) is an element of L+


and
Z
Z
|fn |d = |f |d.
lim
n

Note:
For f L, the sequence {fn }n1 as defined in exercise 4.27 is called
the truncation sequence of f. The truncation sequence is useful
in proving results about functions in the class L.
(4.28) Let f L and
(E) := {x X | f (x) E}, E BR .
Show that is a measure on (R, BR ). Further, if g : R R is any
nonnegative BR -measurable function, i.e., g 1 (A) BR A BR ,
then g f L and
Z
Z
g d =
(g f ) d.
The measure is usually denoted by f 1 and is called the distribution of the measurable function f.

4.3. Intrinsic characterization of nonnegative measurable functions

49

(4.29) Let f L+ be a bounded function, say f (x) N x X and for


some N N. Show that


Z
N
2n
X
k1
k
k1
x|
f (x) < n .
(4.1)
f d = lim
n
2n
2n
2
k=1

m
a

Figure: Area below the curve y = f (x) from inside


Note that here, for every n N, we are taking the partition
{0, 1/2n , 2/2n , . . . , N } of the range of f and using it to induce the
measurable partition Pn of the domain |X| of f , given by
{Xkn | 1 k N 2n }, where Xkn := f 1 [(k 1)/2n , k/2n ).
Using this partition Pn , we form the sums on the right hand side
of (4.1), approximating the area below the curve y = f (x) from
inside (see Figure). Thus in our integral we use partitions of the
range, whereas in Riemann integration we use partitions of the
domain. In the words of H. Lebesgue, the situation is similar to
the problem of finding the total amount in a cash box containing
say n coins, the ith coin being of denomination i , 1 i n.
One method of finding the total P
amount in the box is to add the
value of each coin, i.e., the sum ni=1 i . The other method is to
separate out coins of similar denominations. IfPthere are kj coins
of denominations j , then the total amount is j kj j .
) be its completion.
(4.30) Let (X, S, ) be a measure space and let (X, S,

function. Show that there


Let f : X R be an S-measurable
exists an S-measurable function f : X R such that f (x) = f (x)
for a.e. x().
(4.31) Let (X, S, ) be a complete measure space and {fn }n1 be a sequence of S-measurable functions on X. Let f be a function on

50

4. Integration

X such that f (x) = lim fn (x) for a.e. x(). Show that f is Sn
measurable.

4.4. Integrable functions


Concepts and examples:
R
Given a measure space (X, S, ) in section 4.2, we defined f d, the integral
for functions f L+ , i.e., f : X R , f nonnegative measurable. If f
+

is measurable, but not necessarily nonnegative, we can


R +write f =
R f f .
+

Since both f and f are nonnegative measurable, f d and f d are


defined. Thus it is reasonableR (as the integral
is Rexpected to be linear) to
R
define the integralR of f to beR f d := f + d f d. The problem can
arise in the case f d = f + d = +. To overcome this difficulty, we
introduce the following definition:
4.4.1. Definition:
f : X R is said to be -integrable if both
RA measurable
R function
+

f d and f d are finite, and in that case we define the integral of f


to be
Z
Z
Z
+
f d := f d f d.

We denote by L1 (X, S, ) (or simply by L1 (X) or L1 ()) the space of all


-integrable functions on X.
The properties of the space L1 (X, S, ) and of the map f 7
f L1 (X, S, ), are given in the next proposition.

f d,

4.4.2. Proposition:
For f, g L and a, b R, the following hold:
(i) If |f (x)| g(x) for a.e. x() and g L1 (), then f L1 ().
(ii) If f (x) = g(x) for a.e. x() and f L1 (), then g L1 () and
Z
Z
f d = gd.

(iii) If f L1 (), then af L1 () and


Z
Z
(af )d = a f d.

(iv) If f and g L1 (), then f + g L1 () and


Z
Z
Z
(f + g)d = f d + gd.

51

4.4. Integrable functions

(v) If f , g L1 (), then (af + bg) L1 () and


Z
Z
Z
(af + bg)d = a f d + b gd.
4.4.3. Proposition:
Let f L1 (). For every E S, let
Z
(E) := E |f |d and

(E) :=

E f d.

Then the following hold:


(i) If (E) = 0, then (E) = 0.
(ii) If (E) = 0, then (E) = 0.
(iii) lim(E)0 (E) = 0, i.e., given any > 0, there exists > 0 such that
(E) < whenever, for E S, (E) < .
(iv) If (E) = 0 E, then f (x) = 0 for a.e. x() on E.
4.4.4. Remark:
It is easy to see that (iii) in the above proposition implies (ii). In fact (ii)
also implies (iii). To see this, suppose (iii) does not hold. Then there exist
an >S0 and sets En S, n 1, such that (En ) < 2n but (En ) . Let
An =
k=n Ek . Then {An }n1 is a decreasing sequence in S and (An )
(En ) 2n . Thus by theorem 2.3.2,
!

An = lim (An ) = 0.
n=1

On the other hand, (An ) (En ) , , contradicting (ii).

We prove next the most frequently used theorem which allows us to


interchange the operations of integration and limits.
4.4.5. Theorem (Lebesgues dominated convergence theorem):
Let {fn }n1 be a sequence of measurable functions and let g L1 () be
such that n, |fn (x)| g(x) for a.e. x(). Let {fn (x)}n1 converge to
f (x) for a.e. x(). Then the following hold:
(i) f L1 ().
R
R
(ii)
f d = limn fn d.
R
(iii) limn |fn f |d = 0.

We state another version of this theorem, which is applicable to series


of functions.

52

4. Integration

4.4.6. Corollary:
P R
Let {fn }n1 be a sequence
of
functions
in
L
()
such
that
1
n=1 |fn |d
P
< +. Then f (x) := n=1 fn (x) exists for a.e. x(), f L1 () and
Z
Z
X
fn d.
f d =
n=1

The following is a variation of the dominated convergence theorem for


finite measure spaces. See exercise (4.38) also.
4.4.7. Theorem (Bounded convergence):
Let (X, S, ) be a finite measure space and f, f1 , f2 , . . . be measurable functions. Suppose there exists M > 0 such that |fn (x)| M a.e. x() and
fn (x) f (x) a.e. x(). Then f, fn L1 (X, S, ) and
Z
Z
f d = lim
fn d.
n

4.4.8. Notes:
(i) The monotone convergence theorem and the dominated convergence theorem (along with its variations and versions) are the most important theorems
used for the interchange of integrals and limits.
(ii) Simple function technique: This is an important technique (similar
to the -algebra technique) used very often to prove results about integrable
and nonnegative measurable functions. Suppose we want to show that a
certain claim () holds for all integrable functions. Then technique is the
following:
(1) Show that () holds for nonnegative simple measurable functions.
(2) Show that () holds for nonnegative measurable / integrable functions by approximating them by nonnegative simple measurable
functions and using (1).
(3) Show that () holds for integrable functions f, by using (2) and the
fact that for f L1 , f = f + f and both f + , f L1 .
The proof of the next proposition is an illustration of this technique.
4.4.9. Proposition:
Let (X, S, ) be a -finite measure space and f L1 (X, S, ) be nonnegative.
For every E S, let
Z
(E) :=
f d.
E

53

4.4. Integrable functions

Then is a finite measure on S. Further, f g L1 (X, S, ) for every g


L1 (X, S, ), and
Z
Z
f d =
f gd.
We shall see some more applications of the dominated convergence theorem in the remaining sections. In the next section we look at some special
properties of L1 (X, S, ) in the particular case when X = R, S = L, the
-algebra of Lebesgue measurable sets, and = , the Lebesgue measure.
As an application of the dominated convergence theorem, we exhibit the
possibility of interchanging the order of integration and differentiation in
the next theorem.
4.4.10. Theorem:
Let ft L1 () for every t (a, b) R. Let t0 (a, b) be such that for
a.e. x(), t 7 ft (x) is differentiable in a neighborhood U of t0 and there
dft
(x) g(x) for a.e. x() and for
exists a function g L1 () such that
dt
R
every t U. Then (t) := ft (x)d(x) is differentiable at t0 and
 !
Z 
dft

(t0 ) =
d(x).
(x)
dt
t0

Exercises:
(4.32) For f L, prove the following:
(i) f L1 () iff |f | L1 (). Further, in either case

Z
Z


f d
|f |d.

(ii) If f L1 (), then |f (x)| < + for a.e. x().

(4.33) Let (X) < + and let f L be such that |f (x)| M for a.e. x()
and for some M. Show that f L1 ().
(4.34) Let f L1 () and E S. Show that E f L1 (), where
Z
Z
f d :=
E f d.
E

Further, if E, F S are disjoint sets, show that


Z
Z
Z
f d =
f d +
f d.
EF

54

4. Integration

(4.35) Let f L1 () and Ei S,P


i 1,
R be such that Ei Ej = for
i 6= j. Show that the series
i=1 Ei f d is absolutely convergent,
S
and if E := i=1 Ei , then
Z
Z
X
f d =
f d.
i=1

Ei

(4.36) (i) For every > 0 and f L1 (), show that


Z
1
|f |d < .
{x X | |f (x)| }

This is called Chebyshevs inequality.


(ii) Let f L1 (), and let there exist M > 0 such that
Z
1


f d M
(E) E

for every E S with 0 < (E) < . Show that |f (x)| M for
a.e. x().
(4.37) Let be the Lebesgue measure on R, and let f L1 (R, L, ) be
such that
Z
f (t)d(t) = 0, x R.
(,x)

Show that f (x) = 0 for a.e. ()x R.

(4.38) Let I R be an interval and t I, let ft L. Let g L1 ()


be such that t, |ft (x)| g(x) for a.e. x(). Let t0 R be any
accumulation point of I and let f (x) := lim ft (x) exist for a.e. x().
tt0

Then f L1 () and
Z
Z
f d = lim ft (x)d(x).
tt0

Further, if x the Rfunction t 7 ft (x) is continuous, then so is


the function h(t) := ft (x) d, t I.
(Hint: Apply theorem 4.4.5 to every sequence tn t0 .)

(4.39) Let {fn }n1 and {gn }n1 be sequences of measurable functions such
that |fn | gn n. Let f and g be measurable functions such that
lim fn (x) = f (x) for a.e. x() and lim gn (x) = g(x) for a.e. x().
n
n
If
Z
Z
lim
gn d =
g d < +,
n

show that

lim

fn d =

f d.

(Hint: Apply Fatous lemma to (gn fn ) and (gn + fn ).)

4.5. The Lebesgue integral and its relation with the Riemann integral

55

R

(4.40) Let {fn }n0 be a sequence in L1 (X, S, ). Show that
|fn |d n1
R

R
converges to |f0 |d iff
|fn f0 |d n1 converges to zero.
(Hint: Use exercise 4.38.)

(4.41) Let (X, S) be a measurable space and f : X R be S-measurable.


Prove the following:
(i) S0 := {f 1 (E) | E BR } is the -algebra of subsets of X, and
S0 S.
(ii) If : R R is Borel measurable, i.e., 1 (E) BR E
BR , then f is an S0 -measurable function on X.
(iii) If : X R is any S0 -measurable function, then there exists
a Borel measurable function : R R such that = f .
(Hint: Use the simple function technique and note that if is
a simple S0 -measurable function, then
n
X
=
an f 1 (Ei )
i=1

for some positive integer n, ai R for each i, and Ei BR ,


then
n
X
ai (Ei f ).
=
i=1

(4.42) Let {fn }n1 be a decreasing sequence of nonnegative functions in


L1 () such that fn (x) f (x). Show that
Z
fn d = 0 iff f (x) = 0 a.e. x().
lim
x

(4.43) Let , be as in proposition 4.4.9. Let S denote the -algebra of


all -measurable subsets of X. Prove the following:
(i) S S .
(ii) There exist examples such that S is a proper subclass of S .
Show that S = S if {x X | f (x) = 0} = 0.
(4.44) Let (X, S, ) be a finite measure space and {fn }n1 be a sequence
in L1 () such that fn f uniformly. Show that f L1 () and
Z
lim
|fn f | d = 0.
n

Can the condition of (X) < + be dropped?

4.5. The Lebesgue integral and its relation with


the Riemann integral
Concepts and examples:

56

4. Integration

In this section we analyze the integral, as constructed in the previous section,


for the particular situation when X = R, S = L (the -algebra of Lebesgue
measurable sets) and = , the Lebesgue measure. The space L1 (R, L, ),
also denoted by L1 (R)R or L1 (), is called the space of Lebesgue integrable
functions on R, and f d is called the Lebesgue integral of f. For any
set E L, we write L1 (E) for the space of integrable functions on the
measure space (E, L E, ), where is restricted to L E. In the special
case when E = [a, b], we would like to show that the new notion of integral
for f L1 [a, b] indeed extends the notion of Riemann integral. To be precise,
we have the following theorem:
4.5.1. Theorem:
Let f : [a, b] R be a Riemann integrable function. Then f L1 [a, b] and
Z
Z b
f d =
f (x)dx.
a

4.5.2. Remark:
In fact, the proof of the above theorem includes a proof of the following: If
f R[a, b], then f is continuous a.e. x(). This is because
f (x) = lim n (x) = lim n (x) a.e. x().
n

Thus, if we put
E := {x [a, b] | f (x) = lim n (x) = lim n (x)},
n

then E is a Lebesgue measurable set and ([a, b] \ E) = 0. For x E, given


an arbitrary > 0, we can choose n0 such that
n0 (x) n0 (x) < .

(4.2)

Further, if x is not a point in any partition Pn , then we can choose >


0 such that whenever y [a, b] and |x y| < , then y belongs to the
same subinterval of the partition Pn0 to which x belongs. Thus by (4.2),
|f (x) f (y)| < , showing that x is a point of continuity of f . Thus the
set of discontinuity points of f forms a subset of ([a, b]) \ E) P, where
P is the set of partition points of Pn , n = 1, 2, . . . . Hence f is continuous
almost everywhere. Another proof of the converse is given in the one of the
exercises.

Exercises:
(4.45) Let f : [a, b] R be bounded and continuous for a.e. x().

4.5. The Lebesgue integral and its relation with the Riemann integral

57

(i) Let {Pn }n1 be any sequence of partitions of [a, b] such that
each Pn+1 is a refinement of Pn and kPn k 0 as n .
Let n , n be as constructed in theorem 4.5.1. Let x (a, b)
be a point of continuity of f . Show that
lim n (x) = f (x) = lim n (x).

(ii) Using (i) and the dominated convergence theorem, deduce that
f L1 ([a, b]) and
Z
Z
Z
n d.
n d = lim
f d = lim
n

(iii) Show that f R[a, b] and


Z
Z b
f (x)dx.
f d =
a

(4.46) Let f : [0, 1] [0, ) be Riemann integrable on [, 1] for all > 0.


R1
Show that f L1 [0, 1] iff lim0 f (x)dx exists, and in that case
Z 1
Z
f (x)dx.
f (x)d(x) = lim
0

(4.47) Let f (x) = 1/xp if 0 < x 1, and f (0) = 0. Find necessary and sufR1
ficient condition on p such that f L1 [0, 1]. Compute 0 f (x)d(x)
in that case.
(Hint: Use exercise 4.46.)

(4.48) (Mean value property):Let f : [a, b] R be a continuous


function and let E [a, b], E L, be such that (E) > 0. Show
that there exists a real number such that
Z
f (x)(x) = (E).
E

(4.49) Let f L1 (R), and let g : R R be a measurable function such


that g(x) for a.e. x(). Show that f g L1 (R) and there
exists [, ] such that
Z
Z
|f |gd = |f |d.
(4.50) Let f L1 (R, L, ) and let a R be fixed. Define
R

[a,x] f (t) d(t) for x a,


F (x) :=
R

for x a.
[x,a] f (t)d(t)
Show that F is continuous.

58

4. Integration

(Hint: Without loss of generality take f 0 and show that F is


continuous from the left and right. In fact, F is actually uniformly
continuous.)
(4.51) Let f L1 (R, L, ) and let c R be a point of continuity of f.
Show that
Z
f (x) d(x) = f (c).
lim n
n
[c, c + 1/n]
(4.52) (Arzelas theorem):
Let {fn }n1 be a sequence of Riemann integrable functions on [a, b]
such that for some M > 0, |fn (x)| < M x [a, b] and n =
1, 2, . . . . Let fn (x) f (x) x [a, b] and let f be Riemann
integrable on [a, b]. Then
Z b
Z b
lim
fn (x)dx =
f (x)dx.
n a

(4.53) Let f : [a, b] R be any constant function. Show that f


L1 [a, b].
(4.54) Let f : [a, b] R be any bounded measurable function. Show
that f L1 [a, b].
(4.55) Let f : [a, b] R be any continuous function. Show that f
L1 [a, b].
R
(4.56) Let f L1 (R) be such that K f d = 0 for every compact set
K R. Show that f (x) = 0 for a.e. x().

(4.57) Let {fn }n1 be a decreasing sequence


functions in
P of nonnegative
n1
C(a, b) and let f1 L1 (a, b). If n=1 (1)
fn C(a, b), show
that
!
Z b

Z b X


X
n1
n1
(1)
(1)
fn (x) dx =
fn (x)dx
.
a

(Hint: For every n,

n=1

n=1

Pn

k
k=1 (1) fk

f L1 (a, b)).

(4.58) Give examples to show that analogues of the monotone convergence


theorem and the dominated convergence theorem do not hold for
the Riemann integral.
(4.59) Let f L1 (R) and, t [0, ),
Z
g(t) = sup
|f (x + y) f (x)| d(x)

ty t .

Show that g is continuous at t = 0.


(Hint: Use the simple function technique).

4.6. L1 [a, b] as the completion of R[a, b]

59

4.6. L1 [a, b] as the completion of R[a, b]


Concepts and examples:
Recall that, R[a, b] is not a complete metric space under the L1 -metric
Z b

d(f, g) :=
|f (x) g(x)|dx,
a

for f, g R[a, b]. Since every metric space has a completion, the question
arises: what is the completion of R[a, b] under this metric? You might have
seen the abstract construction of the completion of a metric space. We shall
show that for R[a, b] this completion is nothing but L1 [a, b]. Thus L1 [a, b]
is the concrete realization of the completion of R[a, b]. We shall first show
that L1 [a, b] is a complete metric space, and then show that R[a, b] is dense
in L1 [a, b].
4.6.1. Definition:
For f, g L1 [a, b], we say f is equivalent to g, and write f g, if the set
{x [a, b] | f (x) 6= g(x)} has Lebesgue measure zero.
It is easy to see that the relation is an equivalence relation on L1 [a, b].
We denote the set of equivalence classes again by L1 [a, b]. In other words,
we identify two functions f, g with each other if f g. Thus, for g, f
L1 [a, b], f = g iff f (x) = g(x) for a.e. x(), as functions.
4.6.2. Definition:
For f L1 [a, b], we define
kf k1 :=

|f (x)|d(x).

Clearly kf k1 is well-defined, and it is easy to check that the function f 7


kf k1 , f L1 [a, b], has the following properties:
(i) kf k1 0 f L1 [a, b].
(ii) kf k1 = 0 iff f = 0.
(iii) kaf k1 = |a| kf k1 a R and f L1 [a, b].
(iv) kf + gk1 kf k1 + kgk1 f, g L1 [a, b].
The function k k1 is called a norm on L1 [a, b]. (Geometrically it is the
distance of the vector f L1 [a, b] from the vector 0 L1 [a, b].) For
f, g L1 [a, b], if we define
d(f, g) := kf gk1 ,

60

4. Integration

then d is a metric on L1 [a, b], called the L1 -metric . The most important
property of this metric is given by the next theorem.
4.6.3. Theorem (Riesz-Fischer):
L1 [a, b] is a complete metric space in the L1 -metric.
We show next that C[a, b], the space of continuous function on [a, b],
(and hence R[a, b]) is dense in L1 [a, b].
4.6.4. Theorem:
The space C[a, b] is a dense subset of L1 [a, b].
Since C[a, b] R[a, b], theorems 4.6.3 and 4.6.4 together prove the following theorem:
4.6.5. Theorem:
L1 [a, b] is the completion of R[a, b].
4.6.6. Notes:
(i) In the proof of theorem 4.6.3, one does not use require anywhere the
fact that the functions are defined on an interval. One can define k k1
for functions defined on (E, L E, ), where E L is arbitrary, and the
proof of theorem 5.6.1 will show that L1 (E) := L1 (E, L E, ) is also a
complete metric space under the metric
Z
kf gk1 :=
|f (x) g(x)|d(x).
E

A closer look will show that the metric d makes sense on L1 (X, S, ),
where (X, S, ) is any complete measure space and we identify functions
which agree for a.e. (). Further, theorem 4.6.3 also remains true for
L1 (X, S, ).
(ii) Parts of the proof of theorem 4.6.4 also exhibit the following facts which
are of independent interest. Let f L1 [a, b] and > 0 be given. Then
there exists a simple function such that kf k1 < and a step function
h such that kf hk1 < . Thus simple functions in L1 [a, b] are dense in it.

Exercises:
(4.60) Let f L1 (R) and let > 0 be given. Prove the following:
(i) There exists a positive integer n such that
kf [n,n] f k1 < .

4.6. L1 [a, b] as the completion of R[a, b]

61

(ii) There exists a continuous function g on R such that g is zero


outside some finite interval and kf [n,n] gk1 < .
(iii) For f : R R, let
supp (f ) := closure {x R | f (x) 6= 0}.
The set supp(f ) is called the support of f and is the smallest
closed subset of R outside which f vanishes. Let the space of
continuous functions on R be denoted by C(R) and let
Cc (R) := {f : R R | f C(R) and supp(f ) is compact}.
Then Cc (R) is a dense subset of L1 (R).
(4.61) Let fR L1 (R) and fR(x) := f (x) x R. Prove the following:
(i) f(x)d(x) = f (x)d(x).
(ii) f g L1 (R) for any bounded measurable function g : R R.
Dense subspaces of L1 (R) are very useful in proving results
about L1 -functions. As an application of exercise 4.59 we have the
following:
(4.62) Let f L1 (R) and for every h, k R, let
fh (x) := f (x + h) and (x) := f (kx + h), x R.
Prove the following:
(i) fh , L1 (R) with
Z
Z
Z
Z
(x)d(x) = |k| f (x)d(x) and fh (x)d(x) = f (x)d(x).
(ii) For every h R, fh L1 (R) and lim kfh f k1 = 0.
h0

(Hint : Use exercise 4.60 to get a function g Cc (R) with


kg f k1 < , and note that kgh gk1 < 2(b a + 1) when
supp(g) [a, b] and |h| is sufficiently small.)
(iii) The function h 7 kfh k1 is continuous.
(4.63) (Riemann-Lebesgue lemma): Let f L1 (R), and let g : R
R be any bounded measurable function such that g(x+p)g(x) = 0
for every x R and p R fixed. Show that t 6= 0,
Z



f (x)g(tx)dx M kfp/t f k1 ,


where M = sup |g(x)|/2. Hence deduce (using exercise 4.60) that
Z
lim
f (x)g(tx)dx = 0.
|t|

62

4. Integration

(In the special case when f L1 [0, 2] and g(x) = cos x or sin x,
we have
Z 2
Z 2
lim
f (x) cos nxdx = 0 = lim
f (x) sin nxdx.
n 0

n 0

This finds applications in the theory of Fourier series.)

Chapter 5

Measure and
integration on product
spaces

5.1. Introduction
Concepts and examples:
The intuitive notion of length, originally defined for intervals in R, leads one
to the class of Lebesgue measurable sets which included not only the intervals
and all the topologically nice subsets of R, but also BR the -algebra of
Borel subsets of R. In a similar manner, one would like to extend the notion
of area in R2 (volume in R3 , and so on) to a larger class of subsets which
includes BR2 (BR3 ) the -algebra generated by open subsets of R2 (R3 ).
In the abstract setting, given measure spaces (X, A, ) and (Y, B, ), one
would like to define a measure on the -algebra generated by sets of the
form {A B | A A, B B} in the natural way: (A B) = (A)(B).
We call this natural for, when X = Y = R, A = B = BR and = = , the
Lebesgue measure, then for intervals I and J, (I J) = (I)(J) is the area
of the rectangle with sides the intervals I and J. Thus will automatically
be an extension of the notion of area in R2 . We note that the collection
R := {A B | A A, B B} is only a semi-algebra of subsets of A B in
general. Let A B denote the -algebra of subsets of X Y generated by
R.

63

64

5. Measure and integration on product spaces

5.1.1. Definition:
Let (X, A) and (Y, B) be measurable spaces. A subset E XY is called
a measurable rectangle if E = AB for some A A and B B. We
denote by R the class of all measurable rectangles. The -algebra of subsets
of XY generated by the semi-algebra R is called the product -algebra
and is denoted by AB.
5.1.2. Proposition:
Let pX : X Y X and pY : X Y Y be defined by
pX (x, y) = x

and

pY (x, y) = y,

x X, y Y . Then the following hold:


(i) The maps pX and pY are measurable, i.e., A A, B B we
have p1
(A) A B and p1
(B) A B.
X
Y
(ii) The -algebra A B is the smallest -algebra of subsets of X Y
such that (i) holds.
5.1.3. Proposition:
Let X and Y be nonempty sets and let C, D be families of subsets of X and
Y, respectively. Let C D := {C D | C C, D D}. Then the following
hold:
(i) S(C D) S(C) S(D).
(ii) Let C and D have the property that there exist increasing sequences
{Ci }i1 and {Di }i1 in C and D respectively such that

i=1

Then

Ci = X and

Di = Y .

i=1

S(C D) = S(C) S(D).

Exercises:
(5.1) Let (X, A) be a measurable space. Let , R and E A BR .
Show that {(x, t) X R | (x, t + ) E} A BR .
(Hint: Use the -algebra technique.)
(5.2) Let E BR . Show that {(x, y) R2 | x + y E} and {(x, y)
R2 | x y E} are elements of BR BR .
(5.3) Let X and Y be nonempty sets and C, D be nonempty families of
subsets of X and Y , respectively, as in proposition 5.1.3. Is it true
that S(CD) = S(C)S(D) in general? Check in the case when

65

5.2. Product of measure spaces

C = {} and D is a -algebra of subsets of Y containing at least


four elements.
(5.4) Let BR2 denote the -algebra of Borel subsets of R2 , i.e., the algebra generated by the open subsets of R2 . Show that
BR2 = BR BR .
(Hint: Use proposition 5.1.3.)

5.2. Product of measure spaces


Concepts and examples:
For the rest of the chapter, let (X, A, ) and (Y, B, ) be fixed measure
spaces. Subsets of X Y of the form A B, A A, B B, are called
measurable rectangles. As before, let R denote the class of all measurable
rectangles. The -algebra of subsets of X Y generated by R, denoted by
A B, is called the product -algebra.
The problem we want to analyze in this section is the following: how
can we construct a measure : A B [0, +] such that (A B) =
(A)(B) for every A A, B B? The answer is given by the next theorem.
5.2.1. Theorem:
Let : R [0, ] be defined by
(A B) := (A)(B),

A A, B B.

Then is a well-defined measure on R. Further, if , are -finite, then


there exists a unique measure : A B [0, +] such that
(A B) = (A B) for every A B R.

5.2.2. Definition:
The measure on A B given by theorem 5.2.1 is called the product
of the measures and and is denoted by . The measure space
(X Y, A B, ) is called the product of the measure space (X, A, )
and (Y, B, ), or just the product measure space.
We note that is uniquely defined on A B when , are -finite.
So from now on we shall assume that and are -finite. As is clear,
is defined on AB via the extension theory . So, the natural question arises:
how to compute ( )(E) for a general element E A B?

66

5. Measure and integration on product spaces

5.2.3. Definition:
Let E X Y, x X and y Y . Let
Ex := {y Y | (x, y) E}

and

E y := {x X | (x, y) E}.

The set Ex is called the section of E at x (or x-section of E) and the


set E y is called the section of E at y (or y-section of E).
5.2.4. Examples:
(i) Let E = A B, where A A and B B. Clearly, Ex = B if x A and
Ex = if x 6 A. Similarly, E y = A if y B and E y = if y 6 B.
(ii) Let (X, A) be a measurable space and let A A. Let
E = {(x, t) X R | 0 t < A (x)}.
It is easy to see that
E = (A [0, 1)) (Ac {0}).
Thus
Ex =
Similarly,

[0, 1)
{0}

if x A,
if x
6 A.

X if y = 0,
y
A if y (0, 1),
E =

if y
6 [0, 1).

5.2.5. Proposition:
For E, F, Ei A B and i I, any indexing set, the following hold x
X, y Y :
S
S
S
S
(i) ( iI Ei )x = iI (Ei )x and ( iI Ei )y = iI (Ei )y .
T
T
T
T
(ii) ( iI Ei )x = iI (Ei )x and ( iI Ei )y = iI (Ei )y .
(iii) (E \ F )x = Ex \ Fx and (E \ F )y = E y \ F y .
(iv) If E F, then Ex Fx and E y F y .

5.2.6. Theorem:
Let E A B. Then the following hold:
(i) Ex B and E y A for every x X, y Y.
(ii) The functions x 7 (Ex ) and y 7 (E y ) are measurable functions on X and Y, respectively.
(iii)
Z

(Ex )d(x) = ( )(E) =

(E y )d(y).

67

5.2. Product of measure spaces

5.2.7. Remark:
Given -finite measure spaces (X, A, ) and (Y, B, ), we showed that is
the unique measure on AB such that ()(AB) = (A)(B). Note that
the measure space (X Y, AB, ) need not be a complete measure space
even if the measure spaces (X, A, ) and (Y, B, ) are complete. For example,
if A X, A 6 A and =
6 B B with (B) = 0, then ( ) (A B) = 0,
but A B 6 A B. In fact, theorem 5.2.1 itself gives a complete measure
space (X Y, A B, ), where A B is the -algebra of -measurable
subsets of X Y , being as in theorem 5.2.1, and is the restriction
of to A B. The measure space (X Y, A B, ) is nothing but
the completion of the measure space (X Y, A B, ). It is easy to see
that theorem 5.2.6 holds for E A B also, as claimed in the next exercise
(5.7).
Exercises:
(5.5) Let (X, A, ) be a -finite measure space. For any nonnegative
function f : X R, let
E (f ) := {(x, t) X R | 0 t f (x)}
and
E (f ) := {(x, t) X R | 0 t < f (x)}.
Then the following hold:
(i) If {fn }n1 is an increasing sequence of nonnegative functions
on X increasing to f (x), show that {E (fn )}n1 is an increasing sequence of sets in ABR with

E (fn ) = E (f ).

n=1

(ii) Using (i) and the simple function technique, show that E (f )
A BR , whenever f : X R is a nonnegative measurable
function. Deduce
that E (f ) A BR .
T

(Hint: E (f ) = n=1 E (f + 1/n).)


(iii) Let f : X R be a nonnegative function such that E (f )
A BR . Using exercise (5.1) and the following equality
A := {(x, t) X R | f (x) > c, t > 0}

[
{(x, t) X R | (x, t/n + c) E (f ), t > 0},
=
n=1

show that A A BR , and deduce that f is measurable.

68

5. Measure and integration on product spaces

(iv) Let f : X R be any measurable function. Show that


G(f ) A BR , where
G(f ) := {(x, t) X R | f (x) = t}.
The set G(f ) is called the graph of the function.
(v) Let f L1 (X, A, ), where (X) < +. Show that
Z

( )(E |f |) =
|f |d = ( )(E |f |).

(5.1)

(Hint: First prove this for the case when f is bounded. For the
general case, consider fn = |f | n and note that {E (fn )}n1
increases to E |f |.)
(vi) Extend (iv) to the case when is -finite.
R
The identity (5.1) shows that for nonnegative functions f d
represents the area below the curve y = f (x).
(5.6) Let X be any nonempty set and P(X) be its power set. Let A :=
P(X) P(X) and let D := {(x, y) X X | x = y}. Suppose
D A. Prove the following statements:
(i) There exist sets Ai , Bi P(X) such that D belongs to the
-algebra generated by (Ai Bi ), i = 1, 2, . . . .
(Hint: See exercise (1.19).)
(ii) Let B = S({Ai | i = 1, 2, . . .}). Then card (B) c. For every
x, y X, Dx 6= Dy if x 6= y and Dx B x X. Deduce
that card (X) c.
(iii) If card (X) > c, then D 6 P(X) P(X), even though Dx
P(X) and Dy P(X), x, y X. Hence in general,
P(X) P(X) 6= P(X X).
(5.7) Let E A B. Then the functions x 7 (Ex ) and y 7 (E y )
are measurable and
Z
Z
(E y )d(y).
(Ex )d(x) = ( )(E) =
X

(Hint: E = F N, F A B and ( ) (N ) = 0 by theorem


3.4.7.)

(5.8) Let E AB be such that (E y ) = 0 for a.e. ()y Y . Show that


(Ex ) = 0 for a.e. ()x X. What can you say about ( )(E)?

5.3. Integration on product spaces: Fubinis theorems

69

5.3. Integration on product spaces: Fubinis


theorems
Concepts and examples:
Let (X, A, ) and (Y, B, ) be -finite measure spaces and (XY, AB, )
the product measure space. Theorem 5.2.6 can be interpreted as follows: for
every E A B,
Z

XY

Z Z


E (x, y)d( )(x, y) =
E (x, y)d(y) d(x)
Y
X


Z
Z
=
E (x, y)d(x) d(y).
Y

This allows us to compute the integral of the function E (x, y) by integrating


one variable at a time. So, the natural question arises: does the above hold
when E is replaced by a nonnegative measurable function on X Y ? The
answer is yes, and is made precise in the next theorem.
5.3.1. Theorem (Fubini):
Let f : X Y R be a nonnegative A B-measurable function. Then
the following statements hold:
(i) For x0 X and y0 Y fixed, the functions x 7 f (x, y0 ) and
y 7 f (x0 , y) are measurable on X and Y, respectively.
R
R
(ii) The functions y 7 X f (x, y)d(x) and x 7 Y f (x, y)d(y) are
well-defined nonnegative measurable functions on Y and X, respectively.


Z Z
Z Z
(iii)
f (x, y)d(y) d(x) =
f (x, y)d(x) d(y)
Y
X Z
X
Y
=
f (x, y)d( )(x, y).
XY

In view of theorem 5.3.1, it is natural to expect a similar result for


f L1 ( ), and being -finite measures. This is given by the
following theorem.
5.3.2. Theorem (Fubini):
Let f L1 ( ). Then the following statements are true:
(i) The functions x 7 f (x, y) and y 7 f (x, y) are integrable for
a.e. y() and for a.e. x(), respectively.

70

5. Measure and integration on product spaces

(ii) The functions


Z
Z
y 7
f (x, y)d(x) and x 7
f (x, y)d(y)
X

are defined for a.e. y() and a.e. x() , and are , -integrable,
respectively.

Z Z
Z
(iii)
f (x, y)d(x) d(y) =
f (x, y) d( )
X
Y
XYZ Z

=
f (x, y)d(y) d(x).
X

Theorem 5.3.1, exercise (5.9) and theorem 5.3.2 together give us the
following theorem, which enables us to check the integrability of a function
of two variables and compute its integral.
5.3.4. Theorem:
Let (X, A, ) and (X, B, ) be -finite measure spaces. Let f : X Y R
be an AB-measurable function such that f satisfies any one of the following
statements:
(i) f is nonnegative.
(ii) f L1 ( ).

Z Z
(iii)
|f (x, y)|d(y) d(x) < +.
Y

(iv)

Z Z
Y


|f (x, y)|d(x) d(y) < +.

Then
Z

XY

f (x, y)d( ) =

Z Z
X


f (x, y)d(y) d(x)
Y

Z Z
f (x, y)d(x) d(y),
=
Y

in the sense that all the integrals exist and are equal.
We give next some examples which illustrate the necessity of the conditions on , and f for the conclusions of theorem 5.3.4 to hold.
5.3.5. Example:
Let X = Y = [0, 1] and A = B = B[0,1] , the -algebra of Borel subsets of
[0, 1]. Let be the Lebesgue measure on A and be the counting measure
on B, i.e., (E) := number of elements in E if E is finite and (E) := +

5.3. Integration on product spaces: Fubinis theorems

71

otherwise. Let D := {(x, y) | x = y}. Let, for n 1,


Dn :=

n
[

( [(j 1)/n, j/n] [(j 1)/n, j/n] ).

j=1

Then D =
n=1 Dn . Thus D A B. In fact, D is a closed subset of
[0, 1] [0, 1]. Further,
Z
Z
D (x, y)d(x) = 0 y Y and
D (x, y)d(y) = 1 x X.
X

Hence
Z Z
Y



Z Z
D (x, y)d(y) d(x) = 1.
D (x, y)d(x) d(y) = 0 and
X

This does not contradict theorem 5.3.1, since is not -finite.


5.3.6. Example:
Let X = Y = [0, 1], A = B = B[0,1] , and let = be the Lebesgue measure
on [0,1]. Let

x2 y 2
if (x, y) 6= (0, 0),
f (x, y) :=
(x2 + y 2 )2

0
if x = y otherwise.

Noting that for fixed x, f (x, y) is a Riemann integrable function on [0, 1] and



y
= f (x, y),
y x2 + y 2
it is easy to see that

Z 1Z 1
Z
f (x, y)d(x) d(y) =
0

1Z 1
0


f (x, y)d(y) d(x) = /4.

This does not contradict theorem 5.3.2, because f 6 L1 (X Y ) as



Z
Z 1 Z 1
|f (x, y)|d( ) =
|f (x, y)|d(y) d(x)
[0,1][0,1]

0
1 Z x
1

1
x


|f (x, y)|d(y) d(x)
0
!
Z
/4

cos 2d d(x)

1
d(x) = +.
2x

72

5. Measure and integration on product spaces

*5.3.7. Example:
Assume the continuum hypothesis and let < be the well-order on [0, 1] such
that x [0, 1], {y [0, 1] | y < x} is at most a countable set. Let
E := {(x, y) [0, 1] [0, 1] | x < y}.
Then y fixed and x [0, 1], E (x, y) = 1 if x < y and 0 otherwise.
Hence E (x, y) is the indicator function of a countable set for y [0, 1]
fixed. Thus it is a Borel measurable function. Similarly, for x [0, 1] fixed,
y 7 E (x, y) is Borel measurable, for it is the indicator function of [0, 1]\C,
a countable set. However, E is not Borel measurable on [0, 1][0, 1]. To see
this, note that B[0,1][0,1] = B[0,1] B[0,1] . Thus if E were Borel measurable,
then by theorem 5.3.1 we should have
Z
(x, y) d( )(x, y)
[0,1][0,1]

[0,1]

[0,1]

[0,1]

[0,1]

E (x, y)d(x) d(y)


!

E (x, y)d(y) d(x).

(5.2)

However, it is easy to see that for x, y [0, 1],


Z
Z
E (x, y)d(y) = 1 and
E (x, y)d(x) = 0.
[0,1]

[0,1]

This contradicts (5.2). Hence E is not Borel measurable.


Exercises:
(5.9) Let f : X Y R be A B-measurable. Show that the following
statements are equivalent:
(i) fZ LZ1 ( ) := L1 (X Y, A B, ).
(ii)

(iii)

ZY ZX
X

|f (x, y)| d(x) d(y) < +.



|f (x, y)|d(y) d(x) < +.

(5.10) Let (X, A, ) and (Y, B, ) be complete -finite measure spaces and
let (X Y, A B, ) be the completion of (X Y, A B, ).
Let f : X Y R be any nonnegative extended real valued
A B-measurable function. Show that:
(i) The function x 7 f (x, y) is A-measurable for a.e. y() and
the function y 7 f (x, y) is B-measurable for a.e. x().

5.3. Integration on product spaces: Fubinis theorems

73

R
(ii) The function yR7 X f (x, y)d(x) is B-measurable and the
function x 7 Y f (x, y)d(y) is A-measurable. 
Z
Z
Z
f (x, y)d(y) d(x)
(iii)
f (x, y)d( ) =
X ZY Z
XY

f (x, y)d(x) d(y).
=
X

(5.11) Let X = Y = [1, 1], A = B = B[1,1] , and let = be the


Lebesgue measure on [1, 1]. Let
(
xy
if (x, y) 6= (0, 0),
2
(x + y 2 )2
f (x, y) :=
0
otherwise.
Z

Show that
Z 1

Z
f (x, y)d(y) d(x) = 0 =
1

Z

Can you conclude that



Z
1 Z 1
f (x, y)d(y) d(x) =

1
1


f (x, y)d(x) d(y).

f (x, y)d( )(x, y)?

XY

(5.12) Let f L1 (X, A, ) and g L1 (Y, B, ). Let


(x, y) := f (x)g(y), x X and y Y.
Show that L1 (X Y, A B, ) and
Z
 Z

Z
(x, y)d( ) =
f d
gd .
XY

(5.13) Let f L1 (0, a) and let


Z a
g(x) :=
(f (t)/t)d(t),0 < x a.
x

Show that g L1 (0, a), and compute

g(x)d(x).
0

*(5.14) Let (X, A, ), and (X, B, ) be as in example 5.3.7. Define, for


x, y [0, 1],

1 if x is rational,
f (x, y) :=
2y if y is irrational.
Z

Compute


Z
1 Z 1
f (x, y)d(y) d(x) and
0

Is f in L1 ( )?

1 Z 1
0


f (x, y)d(x) d(y).

74

5. Measure and integration on product spaces

(5.15) Let (X, A, ) be as in example 5.3.7. Let Y = [1, ), B = LR


[1, ), and let be the Lebesgue measure restricted to [1, ).
Define, for (x, y) X Y,
f (x, y) := exy 2e2xy .
Show that f 6 L1 ( ).
(5.16) Let X be a topological space and let BX be the -algebra of Borel
subsets of X. A function f : X R is said to be Borel measurable if f 1 (E) BX E BR . Prove the following:
(i) f is Borel measurable iff f 1 (U ) BX for every open set
U R.
(Hint: Use the -algebra technique.)
(ii) Let f : X R be continuous. Show that f is Borel measurable.
(iii) Let {fn }n1 be a sequence of Borel measurable functions on
X such that f (x) := lim fn (x) exists x X. Show that f
n
also is Borel measurable.
(iv) Consider R2 with the product topology and let f, g be Borel
measurable functions on R. Show that the function on R2
defined by
(x, y) := f (x)g(y),

x X, y Y,

is Borel measurable.
(5.17) Let f : R2 R be Borel measurable. Show that for x X fixed,
y 7 f (x, y) is a Borel measurable function on R. Is the function
x 7 f (x, y), for y Y fixed, also Borel measurable?
(5.18) Let f : R2 R be such that for x X fixed, y
7 f (x, y) is
Borel measurable and for y Y fixed, x 7 f (x, y) is continuous.
(i) For every n 1 and x, y R, define
fn (x, y) := (i nx)f ((i 1)/n, y) + (nx i + 1)f (i/n, y),
whenever x [(i 1)/n, i/n), i Z. Show that each fn : R2
R is continuous and hence is Borel measurable.
(ii) Show that fn (x, y) f (x, y) as n for every (x, y) R2 ,
and hence f is Borel measurable.
(5.19) Let (X, A) and (Y, B) be measurable spaces and let f : X Y R
be a nonnegative A B-measurable function. Let be a -finite
measure on (Y, B). For any E B and x X, let
Z
(x, E) :=
f (x, y)d(y).
E

Show that (x, E) has the following properties:

5.4. Lebesgue measure on R2 and its properties

75

(i) For every fixed E B, 7 (x, E) is an A-measurable function.


(ii) For every fixed x X, E 7 (x, E) is a measure on (Y, B).
A function : X B [0, ) having properties (i) and (ii) above
is called a transition measure.

5.4. Lebesgue measure on R2 and its properties


Concepts and examples:
We now specialize the construction of (X Y, A B, ) to the case when
X = Y = R, A = B = LR and = = , the Lebesgue measure. We
have already seen in exercise 5.4 that BR BR = BR2 and that (R2 , LR
LR , ) is not complete. The completion of this measure space, denoted
by (R2 , LR2 , R2 ), is called the Lebesgue measure space. Elements of
LR2 are called the Lebesgue measurable subsets of R2 , and R2 is called
the Lebesgue measure on R2 . The following proposition ensures that R2
is the unique extension of the natural concept of area in R2 .
5.4.1. Proposition:
Let I denote the collection of left-open, right-closed intervals in R, and let
Then the following hold:
I 2 := {I J | I, J I}.
(i) I 2 is a semi-algebra of subsets of R2 , and S(I 2 ) = BR2 .

(ii) R2 (I J) = (I)(J), I, J I.
(iii) The measure space (R2 , LR2 , R2 ) is the completion of the measure
spaces (R2 , LR LR , ) and (R2 , BR2 , R2 ).
We describe next some properties of the Lebesgue measure R2 . For
E R2 and x R2 , let E + x := {y + x | y E}.
5.4.2. Theorem:
The Lebesgue measure R2 has the following properties:
(i) Let E BR2 and x R2 . Then E + x BR2 and
R2 (E) = R2 (E + x).
(This property of R2 is called translation invariance.)
(ii) For every nonnegative Borel measurable function f on R2 and y
R2 ,
Z
Z
Z
f (x + y) dR2 (x) =
f (x) dR2 (x) =
f (x) dR2 (x).

76

5. Measure and integration on product spaces

(iii) Let be any -finite measure on BR2 such that (E + x) = (E)


E BR2 , x R2 . Suppose
0 < (E0 ) = CR2 (E0 ) < +,
for some E0 B and for some C 0. Then
(E) = CR2 (E), E BR .
5.4.3. Theorem:
Let T : R2 R2 be a linear transformation and E LR2 . Then T (E)
LR2 and
R2 (T (E)) = | det T | R2 (E).
5.4.4. Theorem (Integration of radial functions):
Let f : [0, ) (0, ) be a nonnegative measurable function. Then
Z
Z
f (|x|)dR2 (x) = 2
f (r)rd(r).
R2

Proof: The proof is once again an application of the simple function technique. We outline the steps
Step 1: The theorem holds for f = (a,b) , 0 a < b < +.
Step 2: Let {En }n1 be a sequence of sets from [0, ) LR such that
either the En s are pairwise disjoint, or the En s are increasing. If the
theorem holds for each En , then the theorem holds for E also, where
S
E=
n=1 En .

Step 3: The theorem holds for f = U , U being any open subset of [0, ).
Step 4: The theorem holds for f = N , where N [0, ) and (N ) = 0.
Step 5: The theorem holds when f = E , E LR and E [0, ).
Step 6: The theorem holds for any nonnegative measurable function.

Exercises:
(5.19) (Regularity of R2 ): Prove the following:
(i) R2 (U ) > 0 for every nonempty open subset U of R2 .
(ii) A set E LR2 iff > 0, there exists an open set U such
that E U and (U \ E) < .
(Hint: Proceed as in theorem 3.5.2.)
(iii) R2 (K) < + for every compact subset K of R.

5.4. Lebesgue measure on R2 and its properties

77

(iv) R2 (U ) = sup{R2 (K) | K compact , K U }.


(Hint: Given U open, there exists a sequence of compact sets
Kn such that Kn U .)
(v) If E BR2 is such that R2 (E) < +, then
R2 (E) = sup{R2 (k) | K E, K compact}.
(5.19) Show that for f L1 (R2 , LR2 , R2 ), x R2 , the function y 7
f (x + y) is integrable and
Z
Z
f (x + y)dR2 (x) =
f (x)dR2 (x).
(Hint: Use exercise (4.30) and theorem 5.4.2.)

(5.20) Let E LR2 and x = (x, y) R2 . Let


xE := {(xt, yr) | (t, r) E}.
Prove the following:
(i) xE LR2 for every x R, E LR2 , and R2 (xE) = |xy|R2 (E).
(ii) For every nonnegative Borel measurable function f : R2 R,
Z
Z
f (xt)dR2 (t) = |xy| f (t)dR2 (t),
where for x = (x, y) and t = (s, r), xt := (xs, yr).
(iii) Let R2 {x R2 | |x| 1} =: . Then

R2 {x R2 | |x| < 1} = and R2 {x R2 | |x| < r} = r2 .


(iv) Let E be a vector subspace of R2 . Then R2 (E) = 0 if E has
dimension less than 2.
(5.21) Let T : R2 R be a linear map.
(i) If N R2 is such that R2 (N ) = 0, show that R2 (T (N )) = 0.
(ii) Use (i) above and proposition 5.4.1(ii) to complete the proof
of theorem 5.4.3 for sets E LR2 .
(5.14) Consider the vectors (a1 , b1 ), (a2 , b2 ) R2 and let
P := {(1 a1 + 2 a2 , 1 b1 + 2 b2 ) R2 | 1 , 2 R, 0 i 1},
called the parallelogram determined by these vectors. Show that
R2 (P ) = |a1 b2 a2 b1 |.
(5.15) Let E LR2 with 0 < (E) < . Show that there exists a unique
point (c1 , c2 ) R2 such that
Z
xE (x + c1 , y + c2 )dR2 (x, y) = 0
and

yE (x + c1 , y + c2 )dR2 (x, y) = 0.

78

5. Measure and integration on product spaces

In fact,
c1 =
and

1
(E)

xE (x, y)dR2 (x, y)

Z
1
yE (x, y)dR2 (x, y).
c2 =
(E)
(c = (c1 , c2 ) is called the centroid of E.)
(5.16) Let f : [0, ) R be a measurable function. Show that, if either
of the two integrals in theorem 5.4.4 exists, then so does the other
and the equality holds.
(5.17) Let f : (a, b) (c, d) R be such that
(i) f is continuous;
f
(ii)
exists and is continuous;
x
d
(iii) for some t (a, b), f (t, y) exists y;
dy
 
f
(iv)
exists and is continuous.
y x
 
f
f
and
exist. Moreover,
Show that
y
x y
 
 
f
f
=
.
x y
y x

(Hint: Use Fubinis theorem and the fundamental theorem of calculus to show that (a, b), (c, d) and s < y
 
Z Z
t

f (, ) f (t, ) f (, s) + f (t, s) =
dx dy,
x
s
t y
and note that the inner integral on the right is a continuous function
of .)

Chapter 6

Lp-spaces

Throughout this chapter, we shall work with a fixed -finite measure space
(X, S, ), which is assumed to be complete.
The main aim of this chapter is to analyze the convergence of sequences
of measurable functions. Given a sequence {fn }n1 of measurable functions
on (X, S, ), we have already come across concepts like pointwise convergence of {fn }n1 to a measurable function f, i.e., when {fn (x)} converges
to f (x) x X. When a sequence does not converge pointwise, one would
like to find other methods of analyzing the behavior of the sequence {fn }n1
for large n. Analysis of these methods, and the relations between them, is
the main aim of this chapter. But before we do that, we extend the notion
of measurability and integration to complex-valued functions.

6.1. Integration of complex-valued functions


Concepts and examples:
Let (X, S, ) be a measure space and let C denote the field of complex
numbers. For a function f : X C, consider the functions Re (f ) and
Im (f ) defined by: x X,
Re (f )(x) := Real part of f (x)
and
Im (f )(x) := Imaginary part of f (x).
The functions Re (f ) and Im (f ) are called, respectively, the real part and
the imaginary part of the function f. Note that Re (f ) and Im (f ) are
real-valued functions on X.
79

80

6. Lp -spaces

6.1.1. Definition:
A complex-valued function f : X C is said to be measurable if both
Re (f ) and Im (f ) are measurable functions. We say f is -integrable if
both Re (f ) and Im
R (f ) are integrable. In that case we define the integral
of f, denoted by f d, to be
Z
Z
Z
f d :=
Re (f )d + i Im (f )d.
We denote the set of all complex-valued -integrable functions on X
by L1 (X, S, ) itself. Whenever we restrict ourselves to only real-valued
-integrable functions on X, we shall specify it by Lr1 (X, S, ). Our next
theorem tells us that L1 (X, S, ) is as nice a space as Lr1 (X, S, ) is.
6.1.2. Theorem:
(i) Let f, g L1 (X, S, ) and , C. Then f + g L1 (X, S, )
and
Z
Z
Z
(f + g)d = f d + gd.

(ii) Let f : X C be a measurable function. Then f L1 (X, S, )


iff |f | Lr1 (X, S, ). Further, in either case,
Z
Z




f d
|f |d.

(iii) Let f L1 (X, S, ) and E S. Then E f L1 (X, S, ). We write


Z
Z
f d :=
E f d.
E

If E1 , E2 S and E1 E2 = , then
Z
Z
Z
f d =
f d +
E1 E2

E1

f d.

E2

(iv) Let f L1 (X, S, ), and let {En }n1


P be aR sequence of pairwise
is absolutely
disjoint sets from S. Then the series
n=1 ( En f d)
S
convergent. Also, (E f ) L1 (X, S, ), where E := n=1 En , and
Z
Z
X
f d.
f d =
E

n=1 En

That the integral of complex-valued functions behaves as nicely with


respect to limiting operations as the integral of real-valued functions is the
content of the next theorem.

81

6.1. Integration of complex-valued functions

6.1.3. Theorem (Complex form of Lebesgues dominated convergence):


Let {fn }n1 be a sequence in L1 (X, S, ). Let f (x) := lim fn (x) exist for
n

a.e. x X and let there exist a function g Lr1 (X, S, ) such that n,
|fn (x)| g(x) for a.e. x(). Then f L1 (X, S, ) and
Z
Z
lim
fn d =
f d.
n

6.1.4. Theorem:
Let f L1 (X, S, ), where (X) < . Let S be a closed subset of C such
that E S with (E) > 0,


Z
1
f d S.
(E) E
Then f (x) S for a.e. x X.
Exercises:
(6.1 ) Let f L1 (X, S, ) be such that
Z

Z


f d =
|f | d.


What can you conclude about f ?

(6.2) Let f : X C. Show that f is measurable iff f 1 (E) S for every


Borel set E X.
(6.3) Extend the claim of exercise (4.30) to complex-valued measurable
functions.
(6.4) Let {fn }n1 be a sequence of S-measurable complex-valued functions on (X, S, ). Show that the following statements are equivalent: P
r
(i) P
(
n=1 |fn |) L1 (X, S, ).
R
(ii)
n=1 |fn |d < +.
P
Further, if either of the above is true, then (
n=1 fn ) L1 (X, S, )
and
!
Z
Z

X
X
fn d.
fn d =
n=1

n=1

(6.5) State and prove the Riesz-Fischer theorem (theorem 4.6.3) for complexvalued integrable functions.

82

6. Lp -spaces

(6.6) Let f L1 (X, S, ) and


Z
f d = 0 for every E S.
E

Show that f (x) = 0 for a.e. x() (see proposition 5.4.6).

6.2. Lp -spaces
Concepts and examples:
In section 4.6, we analyzed L1 [a, b], the space of Lebesgue integrable functions on [a, b]. We saw that L1 [a, b] is a vector space over R, and that for
f L1 [a, b], we can define the notion of absolute value of f, i.e., kf k1 . Further, this notion of absolute value can be used to define the L1 -metric on
L1 [a, b] so that L1 [a, b] becomes a complete metric space. In this section
we look at examples of a family of spaces of this type, called Lp -spaces.
Throughout this section, we fix a measure space (X, S, ) which is -finite
and complete.
6.2.1. Definition:
Let 0 < p < . Let Lp () := Lp (X, S, ) denote the space of all complexvalued S-measurable functions on X such that
Z
|f |p d < +.
The space Lp () is called the space of pth -power integrable functions.

6.2.2. Proposition:
The space Lp (X, S, ) is a vector space over C.
6.2.3. Definition:
Let f Lp (X, B, ). Define kf kp , called the pth -norm of f, as follows:
1/p
Z
p
.
|f | d
kf kp :=
Since kf kp = kgkp if f (x) = g(x) for a.e. x(), we treat such f and g as
the same element of Lp (X, B, ). To show that kf kp has the properties of a
metric (as was the case for p = 1 in section 4.6), we need some inequalities.
6.2.4. Lemma:
For nonnegative real numbers a, b and 0 < t < 1, the following inequalities
hold:
(i) at b1t ta + (1 t)b.

6.2. Lp -spaces

(ii) (

83

1
a + b 1/t
)
(a1/t + b1/t ).
2
2

6.2.5. Theorem (H
olders inequality):
Let p > 1 and q > 1 be such that 1/p + 1/q = 1. Let f Lp () and
g Lq (). Then f g L1 () and
1/q
1/p Z
Z
Z
q
p
.
|g| d
|f | d
|f g|d
6.2.6. Corollary:
Let p, q > 1 be real numbers with 1/p + 1/q = 1. Let {an }n1 and {bn }n1
be sequences of complex numbers such that

X
X
p
|bn |q < .
|an | < and
n=1

n=1

Then

n=1 |an bn |

< + and
!

n=1

|an bn |

n=1

|an |

!1/p

In particular, if an = bn = 0 n k, then
!1/p
k
k
X
X
|an bn |
|an |p
n=1

n=1

|bn |

n=1

k
X

|bn |q

n=1

!1/q

!1/q

6.2.7. Note:
In the special case when p = q = 2, H
olders inequality is known as the
Cauchy-Schwarz inequality.
6.2.8. Theorem (Minkowskis inequality):
Let 1 p < and f, g Lp (). Then f + g Lp () and
kf + gkp kf kp + kgkp .

6.2.9. Theorem (Riesz-Fischer):


The space Lp (X, S, ), for 1 p < , is a complete metric space with the
metric defined by
dp (f, g) := kf gkp f, g Lp (X, S, ).

84

6. Lp -spaces

Of course, here we identify f, g Lp (X, S, ) if f (x) = g(x) for a.e.


x().
6.2.10. Note:
In theorem 6.2.9, we showed that Lp (), 1 p < , is a complete metric
space under the metric

dp (f, g) := kf gkp :=

Z

1/p
|f g| d
.
p

It is natural to ask the same question for 0 < p < 1. If 0 < p < 1, define for
f, g Lp ()
Z
dp (f, g) := |f g|p d.
Using the inequality (see exercise (6.7))
1 + tp (1 + t)p t 0,
it is easy to see that dp is a metric on Lp (). Also, proceeding as in theorem
6.2.9, we can show that Lp () is a complete metric space for 0 < p < 1.
In the case 1 p < , f 7 kf kp is a real-valued map on Lp () with the
properties: f, g Lp () and C
(i) kf kp 0, and kf kp = 0 iff f = 0.
(ii) kf kp = ||kf kp .
(iii) kf + gkp kf kp + kgkp , called the triangle inequality.
Such a function can be defined on any vector space, and is called a norm.
A vector space with a norm is called a normed linear space. Thus for
1 p < , the Lp -spaces are examples of normed linear spaces which
are also complete under the metric kf gkp , the metric induced by the
norm. Such normed linear spaces are called Banach spaces. However,
when 0 < p < 1, f 7 kf kp is no longer a norm, as it fails to satisfy the
triangle inequality. To see this, consider a measure space (X, S, ) such that
A, B S with A B = and 0 < (A) < , 0 < (B) < . Let , be
positive real numbers. Then

kA kp =

Z

1/p
|| A (x)d(x)
= ((A))1/p .
p

6.2. Lp -spaces

85

Similarly, kB kp = ((B))1/p . Further,


kA + B kp =

Z

Z

(A + B ) d
p

1/p

( A + B ) d

1/p

= (p (A) + p (B))1/p .

Now, using exercise 6.7(ii) with t = (/)p ((B)/(A)), we have


1/p

 

 p
(B)
(B) 1/p

> 1+
,
1+

(A)

(A)
i.e.,
(1/p (A) + 1/p (B))1/p > ((A))1/p + ((B))1/p ,
i.e.,
kA + B kp > kA kp + kB kp .
This is the reason that Lp -spaces for 0 < p < 1 are not very interesting to
study.
Exercises:
(6.7) Let 0 < t < and 0 < p < 1. Show that
(i) (1 + t)p < 1 + tp .
(ii) (1 + t)1/p > 1 + t1/p .
(6.8) Let 0 < p < 1 and < q < 0 be such that 1/p + 1/q = 1. Let
f, g be positive functions such that f Lp () and g Lq (). Show
that
Z
kf kp kgkq <
f gd.
(Hint: Apply H
olders inequality to (f g)p L1/p and g p Lp/q .)
(6.9) If (X) < and 1 p q < , show that Lq () Lp ().
(Hint: Apply H
olders inequality to |f |p Lq/p () and 1 L(qp)/q ().)
(6.10) Let f Lp (X, S, ), where 1 p < . Show that > 0,
({x X| |f (x)| 0}) kf kp /p .
This is called Chebyshevs inequality for Lp -functions.

86

6. Lp -spaces

6.3. L (X, S, )
Concepts and examples:
6.3.1. Definition:
A measurable function f : X R or C is said to be essentially bounded
if there exists some real number M such that ({x X | |f (x)| > M }) = 0.
We denote by L (X, S, ) the set of all essentially bounded functions.
For f L (X, S, ) define
kf k := inf {M | {x X | |f (x)| > M } = 0}.
We call kf k the essential supremum of f.
6.3.2. Theorem:
Let f, g L (X, S, ) and let , be scalars. Then the following hold:
(i) f L (X, S, ) and
kf k = || kf k .
(ii) (f + g) L (X, B, ) and
kf + gk kf k + kgk .
(iii) kf k = 0 iff f (x) = 0 a.e. x().
(iv) There exists a set E S such that (E) = 0 and
kf k = sup |f (x)|.
xX\E

Thus ({x X| |f (x)| > kf k }) = 0.


(v) kf k = sup{N | ({x X| |f (x)| > N }) > 0}.
6.3.3. Corollary:
For f, g L (X, S, ), let
d (f, g) := kf gk .
Then for f, g, h L (X, S, ), the following hold:
(i) d (f, g) 0, and = 0 iff f (x) = g(x) for a.e. x().
(ii) d (f, g) d (f, h) + d (h, g).
(iii) If {fn }n1 is a sequence in L (X, S, ), then d (fn , f ) 0 as
n iff fn f uniformly a.e.
(iv) If {fn }n1 is a Cauchy sequence in L (X, S, ), i.e., d (fn , fm )
0 as n, m , then h L (X, S, ) such that d (fn , h)
0 as n .

6.4. L2 (X, S, )

87

6.3.4. Remark:
For f, g L (X, S, ), let us write
f g if f (x) = g(x) for a.e. x().
Then is an equivalence relation. We denote the set of equivalence classes
by L (X, B, ) itself. For any element f L (X, S, ), which is in fact
an equivalence class, we can define kf k by choosing any element in the
equivalence class f. Then kf k is well-defined, and L (X, S, ) is a Banach
space under this norm.
Exercises:
(6.11 ) Let f L (X, S, ), where (X, S, ) is a finite measure space.
Prove the following statements:
(i) For 1 p < , f Lp (X, S, ) and kf kp kf k ((X))1/p .
(ii) If N > 0 is such that ({x X| |f (x)| > N }) > 0, show that
N ((X))1/p kf kp .
(iii) Let {pn }n1 be any monotonically increasing sequence of real
numbers such that pn 1 n and {pn }n1 is not bounded
above. Using (i), (ii) above and theorem 6.3.2(v), show that
kf k = lim kf kpn .
n

6.4. L2 (X, S, )
Concepts and examples:
The reason for analyzing the space L2 (X, S, ) in detail is the following.
We have already seen that for 1 p , the spaces Lp (X, S, ) are linear
spaces and have a notion of distance (norm) under which they are complete.
Let us recall, for f L2 (X, S, ), that
1/2
Z
2
.
|f (x)| d(x)
kf k2 :=

If we treat f L2 (X, S, ) as a vector with uncountable components, f (x)


being the xth component, x X, then kf k2 can be viewed as a generalization
of the Euclidean magnitude of a vector in Rn with summation being replaced
by integration. We recall that on Rn , we have the notion of dot product of
vectors, which is related to the magnitude of vectors and helps us to define
the notion of angles on Rn . That such a dot product can also be defined on
L2 (X, S, ) and enables one to do geometry on L2 (X, S, ), is the reason we
discuss this space in detail.

88

6. Lp -spaces

6.4.1. Definition:
For f, g L2 (X, S, ), we define
hf, gi :=

f (x) g(x) d(x),

whenever it exists, where g denote the complex conjugate function: g(x) :=


g(x) x X.
6.4.2. Proposition (Cauchy-Schwarz inequality):
For every f, g L2 (X, S, ), hf, gi is a well-defined scalar and
|hf, gi| kf k2 kgk2 .

The scalar hf, gi is called the inner product of f and g.


6.4.3. Proposition:
For f, g, h L2 (X, S, ) and , C, the following hold:
(i) hf, f i 0, and equality holds iff f = 0.
(ii) hf, gi = hg, f i.
(iii) hf + g, hi = hf, hi + hg, hi.
(iv) hf, g + hi = hf, gi + hf, hi.
(v) kf k2 = hf, f i1/2 .
6.4.4.Note:
In the Cauchy-Schwarz inequality, the equality holds iff f and g are linearly
dependent.
6.4.5. Note:
Given an arbitrary vector space H over the field R (or C), if there is a map
h , i : H H R (or C) having the properties (i) to (iv) as given in
proposition 6.4.3, then it is called an inner product space. On every
inner product space H, it is easy to show that kuk := hu, ui1/2 , u H,
is indeed a norm on H, called the norm induced by the inner product.
Further, the Cauchy-Schwarz inequality holds:
|hu, vi| kuk kvk, u, v H.
This can be proved as follows: For u H, let kuk = (hu, ui)1/2 . If either
u = 0 or v = 0, then clearly |hu, vi| = 0 = kuk kvk. So, let u, v H be

such that u 6= 0 and v 6= 0. Then kuk > 0 and kvk > 0. Let u = u/kuk and

6.4. L2 (X, S, )

89

v = v/kvk. Then hu , u i = 1 = hv , v i and

0 hu hu , v iv , u hu , v iv i

= hu , u i + |hu , v i|2 hv , v i 2|hu , v i|2

= 1 |hu , v i|2 .
Hence

|hu , v i| 1 = ku kkv k,
i.e.,
|hu, vi| kukkvk.
If H is also complete under the norm induced by the inner product, then
H is called a Hilbert space. Thus L2 (X, S, ) is an example of a Hilbert
space. We shall not go into the general theory of Hilbert spaces. We discuss
some results for L2 (X, S, ) which are in fact true, without any change in
the arguments, for general Hilbert spaces also.
6.4.6. Definition:
Let f, g L2 (X, S, ). We say that f and g are orthogonal, and write
f g, if hf, gi = 0. For a subset S of L2 (X, B, ), we write f S if
hf, hi = 0 h S.
6.4.7. Definition:
Let S be a nonempty subset of L2 (X, S, ).
(i) We say S is a subspace of L2 (X, S, ) if , C and f, g
L2 (X, B, ), we have f + g S.
(ii) We say S is a closed subspace if it is closed under the kk2 metric,
i.e., for every sequence {fn }n1 in S with lim kfn f k2 = 0 for
some f L2 (X, S, ), we have f S.

6.4.8. Proposition:
Let S be any nonempty subset of L2 (X, S, ) and let
S := {g L2 (X, S, ) | hf, gi = 0 f S}.
Then S is a closed subspace of L2 (X, S, ).
The set S is called the orthogonal complement of S.
We next prove a result which seems geometrically obvious.

90

6. Lp -spaces

6.4.9. Theorem:
Let f L2 (X, S, ), and let S be a closed subspace of L2 (X, S, ). Let
:= inf {kf gk2 | g S}.
Then there exists a unique function f0 S such that = kf f0 k2 . Further,
if f 6 S then 0 6= (f f0 ) S.
6.4.10. Corollary:
Let S be a proper closed subspace of L2 (X, B, ). Then S 6= {0}.
Next we give two applications of theorem 6.4.9. Our first application
says that if S is any closed subspace of L2 (X, S, ), then L2 (X, S, ) =
S + S := {f + g|f S, g S } with S S = {0}.
6.4.12. Theorem:
Let S1 , S2 be subsets of L2 (X, S, ). Then the following hold:
(i) S1 is a closed subspace of L2 (X, S, ) and S1 S1 {0}. If S1 is
also a subspace, then S1 S1 = {0}.
(ii) S1 S2 if S2 S1 .
(iii) S1 (S1 ) , with equality iff S1 is a closed subspace of L2 (X, S, ).
(iv) If S1 and S2 are closed subspaces and f g f S1 and g S2 ,
then S1 + S2 := {f + g | f S1 , g S2 } is also a closed subspace.
(v) If S1 is a closed subspace, then S1 S1 = {0} and L2 (X, S, ) =
S1 + S1 . Thus every f L2 (X, S, ) can be uniquely expressed as
f = g + h, where g S1 and h S1 .
(This is also expressed as L2 (X, S, ) = S1 S1 , and is called the
projection theorem.)
As our second application of theorem 6.4.9, we characterize all bounded
linear functions on L2 , as defined next.
6.4.12. Definition:
A map T : L2 (X, S, ) C is called a bounded linear functional if it
has the following properties:
(i) For every f, g L2 (X, S, ) and , C,
T (f + g) = T (f ) + T (g).
(ii) There exists a real number M such that
|T (f )| M kf k2 , f L2 (X, S, ).

6.4. L2 (X, S, )

91

6.4.13. Proposition:
Let T : L2 (X, S, ) C be such that f, g L2 (X, S, ) and , C
T (f + g) = T (f ) + T (g).
Then the following are equivalent:
(i) T is bounded, i.e., M R such that
|T (f )| M kf k2 , f L2 (X, S, ).
(ii) T is continuous.
(iii) T is continuous at 0 L2 (X, S, ).
6.4.14. Example:
Let g L2 (X, S, ) be fixed. Define the map
Tg : L2 (X, B, ) C
as follows:
Tg (f ) = hf, gi =

f g d f L2 (X, S, ).

It is easy to see that Tg is linear, i.e.,


Tg (f1 + f2 ) = Tg (f1 ) + Tg (f2 )
, C and f1 , f2 L2 (X, S, ). Also, by the Cauchy-Schwarz inequality,
|Tg (f )| = |hf, gi| kgk2 kf k2 .
Hence Tg is a bounded linear functional on L2 (X, S, ). We note that g
(Ker(Tg )) , where
Ker(Tg ) := {f L2 (X, B, ) | Tg (f ) = 0}.
Ker(Tg ) is called the kernel of Tg . Our next theorem tell us that every
bounded linear functional on L2 (X, S, ) is of this type.
6.4.15. Theorem (Riesz representation):
Let T : L2 (X, B, ) C be a bounded linear functional. Then there is a
unique g0 L2 (X, S, ) such that
T (f ) = hf, g0 i f L2 (X, S, ).

6.4.16. Note:
In theorem 6.4.15, we showed that, given a closed subspace M of L2 (X, S, ),
every element f L2 (X, S, ) can be written uniquely as f = g + h, where
g M and h M . Let us denote g by PM (f ), called the projection of f
onto M . Geometrically, PM (f ) is the unique best approximator of f in M.

92

6. Lp -spaces

The properties of the map PM : L2 (X, S, ) L2 (X, S, ) are given in the


next theorem.
6.4.17. Theorem:
PM : L2 (X, S, ) L2 (X, S, ), as defined in note 6.4.16 above, has the
following properties: f, g L2 (X, S, )
(i) PM is linear and PM (f ) = f whenever f M .
(ii) f PM (f ) M .
(iii) hPM (f ), gi = hf, PM (g)i = hPM (f ), PM (g)i.
(iv) PM (PM (f )) = f.
(v) kf k22 = kPM (f )k22 + kf PM (f )k22 .
(vi) PM is continuous.

Exercises:
(6.12) Let f L2 (X, S, ) be such that f g g L2 (X, S, ). What
can you conclude about f ?
(6.13) Pythagoras identity: Let f, g L2 (X, S, ) and f g. Show
that
kf + gk22 = kf k22 + kgk22 .
(6.14) (Bessels inequality):Let f1 , f2 , . . . , fn be elements of L2 (X, S, )
such that kfi k2 = 1 i and fi fj for i 6= j. Show that
n
X

|hf, fj i|2 kf k22 .

j=1

(Hint: Consider hz, zi, where z = f

Pn

j=1 hf,

fj ifj .)

(6.15) Let {fn }n1 , {gn }n1 be sequences in L2 (X, S, ) such that lim kfn
n

f k2 = 0, lim kgn gk2 = 0 for f, g L2 (X, S, ). Show that


n

hfn , hi hf, hi, hh, gn i hh, gi, hfn , gn i hf, gi


for every h L2 (X, S, ). In other words, the inner product map
h , i : L2 L2 C is continuous in each variable and jointly.
(6.16) (Parallelogram identity): Let f, g L2 (X, S, ). Then
kf + gk22 + kf gk22 = 2kf k22 + 2kgk22 .

6.5. L2 -convergence of Fourier series

93

6.5. L2 -convergence of Fourier series


Concepts and examples:
The problem of investigating the convergence of Fourier series motivated
mathematicians to look for an extension of the integral concept. It is natural to ask the question: did the extended integral, i.e., the Lebesgue integral,
achieve success in this direction? One can say without any doubt that one
of the pioneering applications of the Lebesgue integral lies in the study of
Fourier series. We shall not go into the general theory of Fourier series,
but only prove an important result (the Riesz-Fischer theorem) which has
applications in many branches of mathematics, physics and electrical engineering. For the general theory of Fourier series, one can consult Bhatia [4]
and other references given there.
6.5.1. Definition:
For f Lr1 [, ], let
1
an :=

f (x) cos nx d(x), n = 0, 1, 2, . . . ,

and
1
bn :=

f (x) sin nx d(x), n = 1, 2, . . . .

The scalars an , bn are called the Fourier coefficients of f , and the series

X
a0
+
(an cos nx + bn sin nx)
2
n=1

is called the Fourier series of f. Let


sn (x) :=

n
X
a0
+
(ak cos kx + bn sin kx), x [, ].
2
k=1

The function sn (x) is called the nth -partial sum of the Fourier series of f at
x. The main problem analyzed in the theory of Fourier series is the following:
when does {sn (x)}n1 converge to f (x), x [, ]? This is known as the
pointwise convergence problem of Fourier series. For f L1 [, ] and
x [, ], if lim sn (x) = f (x), we say that f has pointwise representation
n
by its Fourier series at x. The answer to the pointwise convergence problem
is not easy, and, given the nature and scope of this text, we shall not go into
it. For details one can consult any one of the texts listed above. We state
below an important relation between functions and their Fourier coefficients.

94

6. Lp -spaces

6.5.2. Theorem (Uniqueness of the Fourier series):


If f Lr1 [, ] is such that all its Fourier coefficients are zero, then f (x) =
0 for a.e. x, i.e., a function is uniquely determined by its Fourier coefficients.
Though the pointwise representation of a function f Lr1 [, ] by its
Fourier series is undeniably of great intrinsic interest, it has its limitations.
First of all, not every function f Lr1 [, ] can have a pointwise representation one has to put extra conditions on f to get a pointwise representation.
Secondly, f Lr1 [, ] need be defined only a.e. Thus pointwise representation makes sense only a.e. Finally, from the point of view of many applications, pointwise representations have very little utility. Such consideration
motivated mathematicians to look for some other methods of analyzing the
convergence problem. We consider below one such method, namely the convergence of Fourier series in the L2 -metric. For other methods we refer to
the texts cited earlier.
Let f Lr2 [, ]. Using the Cauchy-Schwarz inequality, it follows that
f Lr1 [, ] also. Let an , bn be its Fourier coefficients and let sn (x) be the
nth -partial sum of the Fourier-series of f. In the terminology of the previous
section , if we write
k (x) := cos kx and k (x) := sin kx,
then
ak = hf, k i/ and bk = hf, k i/.
6.5.3. Lemma:
For nonnegative integers n and m, the following relations hold:
hn , m i = 0;
hn , m i = 0 if n 6= m and = if n = m;
hn , m i = 0 if n 6= m and = if n = m.

6.5.4. Theorem (Bessels inequality):


For f Lr2 [, ],

X
1
|a0 |2
+
(|an |2 + |bn |2 ) kf k22 .
2

n=1

The above theorem has a converse, as given in the next theorem.

6.5. L2 -convergence of Fourier series

95

6.5.5. Theorem (Riesz-Fischer):


Let {an }n0 and {bn }n1 be sequences of real numbers such that

X
|a0 |2
+
(|an |2 + |bn |2 ) < +.
2
n=1

Then there exists a unique function f Lr2 [, ] such that an , bn are its
Fourier coefficients.
6.5.6. Corollary ( Parsevals identity):
Let f Lr2 [, ]. Then the Fourier series of f converges to f in the L2 norm, i.e., ksn f k2 0 as n . Further,

X
1
|a0 |2
(|an |2 + |bn |2 ).
kf k22 =
+

2
n=1

6.5.7 Note:
A careful observation of the above arguments will tell the reader that Fourier
coefficients can be defined for Riemann integrable functions also (as was done
historically first), and Bessels inequality remains valid. However, RieszFischer critically uses the fact that Lr2 [, ] is complete under the L2 metric, which is not true for Riemann integrable functions.

96

6. Lp -spaces

Exercises:
(6.17) (Eulers identity):
Show that

X
=0

1
2
=
.
(2 + 1)2
8

Appendix A

Extended real numbers

A.1. Extended real numbers


Let R denote the set of all real numbers and let
R := R {+} {},
where + and are two symbols read as plus infinity and minus
infinity. We extend the algebraic operations and the order relation of R to
R as follows:
(1) For every x R, < x < +.
(2) For every x R,
() + x =
and (+) + x = +;
(+) + (+) = + and () + () = .
(3) For every x R,
x(+) = (+)x = +
x() = ()x =
x(+) = (+)x =
x() = ()x = +




if x > 0,

if x < 0.

Further,
(+)0 = ()0 = 0, ()(+) = () and ()() = ().
Note that the relations + (+) and (+) + () are not defined.
97

98

Appendix A

The set R , also denoted as [, +], with the above properties is


called the set of extended real numbers. The symbol + is also denoted
by , when no confusion arises.
A.2.

sup(A) and inf (A)

For a nonempty set A R , we write sup(A) := + if AR is not bounded


above, and inf(A) := if A R is not bounded below. Thus
sup(A) and inf(A) always exist for every nonempty subset A of R .
A.3. Limits of sequences in R
For {xn }n1 any monotonically increasing sequence in R which is not
bounded above, we say {xn }n1 is convergent to + and write
lim xn = +.

Similarly, if {xn }n1 is a monotonically decreasing sequence which is not


bounded below, we say {xn }n1 is convergent to and write
lim xn = .

Hence
every monotone sequence in R is convergent.
Thus for any sequence {xn }n1 in R , the sequences {supkj xk }j1 and
{inf kj xk }j1 always converge. We write
lim sup xn := lim (sup xk )
n

j kj

and
lim inf xn := lim (inf xk ).
n

j kj

limn sup xn is called the limit superior of the sequence {xn }n1 and
lim inf n xn is called the limit inferior of the sequence {xn }n1 . Note
that
lim inf xn lim sup xn .
n

We say a sequence {xn }n1 is convergent to x R if lim inf n xn =


lim supn xn =: x, say. In that case we write limn xn := x.
A.4. Series in R
P
Let {xk }k1 be a sequence in R suchPthat for every n N, sn := nk=1 xk is

well-defined. We say that the series P


k=1 xk is convergent to x if {sn }n1
is convergent. We write this as x =
k=1 xk , x being called the sum of

99

Extended real numbers

P
P
the series
k=1 xk
k=1 xk . For example, if each xk 0, then the series
is always convergent.

Let {xk }k1 be a sequence in RP


, xn 0 for every n. Let : N N be
any bijective map. Then the series
k=1 x(k) is called an arrangement
P
x
.
of the series
k=1 k
P
P
For every rearrangement , the series
k=1 xk conk=1 x(k) and
verge to the same sum.
P
Clearly,
both the series are convergent in R . Let :=
k=1 xk and :=
P
x
.
To
prove

=
,
in
view
of
the
symmetry,
it
is
enough to prove
k=1 (k)
that . Now
n
m

X
X
X
=
x(k)
xk
xk = ,
k=1

k=1

k=1

where in the the middle term m := max{(1), , (k)}. Hence ,


proving the required claim.

Similar arguments apply to {xn,m }n,m1 , any double-indexed


P sequence of
. For every fixed n, the series
nonnegative elements of R
m=1 xn,m is conP
P
vergent in R .PLet yn :=
x
.
Further,
y
is
also
convergent
in
m=1 n,m
n=1 n
P

R . Let y P
:= n=1 yn . We can also define for all fixed m, zm := n=1 xn,m ,

and z :=
m=1 zm in R . We show that both these processes lead to the
same sum, i.e., y = z, which is written as
!
!

X
X
X
X
xn,m .
xn,m =
m=1

n=1

n=1

m=1

To see this, we note that r, s N,


r X
s
s X

X
X
X
xn,m
xn,m
xn,m .
n=1 m=1

Hence

y :=

m=1 n=1

X
X

xn,m

n=1 m=1

m=1 n=1

X
X

xn,m =: z.

m=1 n=1

The reverse inequality follows from symmetry.

A similar result holds for rearrangement of double-indexed series of nonnegative terms in R :


Let : N NN be any bijective map, and let {xn,m }n1,m1 be
a double-indexed sequence of nonnegative elements of R . Then

X
r=1

x(r) =

X
X

n=1 m=0

xn,m =

X
X

m=1 n=1

xn,m .

Appendix B

Axiom of choice

Let A, B be two sets. One defines AB, the Cartesian product of A and
B, to be the empty set if either A or B or both are empty, and to be the set
of all ordered pairs (a, b), a A, b B, when both A and B are nonempty.
Similarly, for a finite family of nonempty sets A1 , . . . , An , we define their
Cartesian product to be the set
A1 An := {(x1 , x2 , . . . xn ) | xi Ai , i = 1, 2, . . . , n}.
Obviously, A1 An is a nonempty set. We
S can think of A1 An as
the set of all functions f : {1, 2, . . . , n} ni=1 Ai with f (i) = xi Ai for
each i. One can copy this to define the Cartesian product of any arbitrary
family of sets, say {A }I , to be the set
)
(
Y
[
A := f : I
A f () A , I .
I

Q
i.e.,we do not
However, there is no surety that the set I A is nonempty,S
know that there always exists at least one function f : I I A such
that f () A I, although intuitively it seems obvious that such a
function should always exist. However, this cannot be proved with the usual
axioms of set theory. (For a short introduction to axiomatic set theory, see
Rana [30]. For detailed account of axiomatic set theory, the axiom of choice,
and its history, see Halmos [15] and Fraenkel [12]). The way out of the above
dilemma is to treat this an axiom itself, called the axiom of choice:
If {A }I is a non-empty family of sets such that
S each A is nonempty, then there exists a function f : I I A such that
f () A I.
101

102

Appendix B

Such a function is called a choice function.


The axiom of choice has many equivalent formulations; a useful one is
the following:
If {A | I} is a nonempty family of pairwise disjoint sets
S such
that A 6= for every I, then there exists a set E I A
such that E A consists of precisely one element for each I.

The axiom of choice finds applications in many diverse branches of mathematics. (We have used it in section 4.6 to construct nonmeasurable subsets
of R.)

Appendix C

Continuum hypothesis

Let X and Y be two sets. We say X and Y are equipotent iff there exists
a bijection between them. We write this as X Y. In a sense, equipotent
sets have same number of elements.
We say a set A is finite if A {1, 2, . . . , n} for some n N, and we say
A is countable if A N, the set of natural numbers. A set which is not
countable is called uncountable. For example, N, Z, Q are all countable
sets while R is uncountable. (For a detailed discussion, see Rana [30].)
The statement X Y means X and Y have the same number of elements, and can be made precise as follows (see Halmos [15] for details). Let
C be a collection of sets such that any two members of C are equipotent to
each other. Then one can assign a symbol, called its cardinal number,
to each A C, denoted by card(A). Thus
card(A) = card(B) iff A B.
The cardinal number of a set A is also called the cardinality of A. For example, for any set A which is equipotent to {1, 2, . . . , n}, we write card(A) = n.
For any set A N, we write card(A) = 0 , called aleph-nought. For any
set A R, we write card(A) = c, called cardinality of the continuum.
For finite sets, it is easy to see that if card(A) = n, then card (P(A)) = 2n ,
where P(A) is the set of all subsets of A. We define, for any nonempty set
X,
card(P(X)) := 2card(X) .
For example,
card(P(N)) := 20 and card(P(R)) := 2c.
103

104

Appendix C

For a finite set A, we know that


card(P(A)) = 2card(A) > card(A).
Can the same be said about arbitrary sets which are not necessarily finite?
For two sets A and B, we say card(A)  card(B) if there exists a one-one
map from A into B. We write card(A) <card(B) if card (A) card(B) but
card(A) 6=card(B). Now we ask the question:
Is card(A) < card(P(A)) for any nonempty set A?
The answer is in the affirmative and is due to George Cantor (see Rana
[30] for a proof). It can be shown that 20 = c, i.e., P(N) R. Thus we
have the following:
0 < 1 < 2 < < n < < 0 < 20 = c < 2c < 22 < .
c

This raises the following natural question:


Does there exist a cardinal number such that 0 < < 20 = c?
That is, does there exist a set A R such that A 6 R and N A but
A 6 N? The answer to this question is not known. The statement that the
answer to the above question is in the negative is called the continuum
hypothesis:
There does not exist any cardinal number between 0 and 20 = c.
An equivalent formulation of this is the following:
The set R can be well-ordered in such a way that each element of
R is preceded by only countably many elements.
We used this in section 3.4, to prove Ulams theorem. It is known that
the continuum hypothesis is independent of the Zermelo-Fraenkel axioms of
set theory.

References

[1]

Aliprantis, C.D. and Burkinshaw, O. Principles of Real Analysis (3rd


Edition). Academic Press, Inc. New York, 1998.

[2]

Apostol, T.M. Mathematical Analysis. Narosa Publishing House, New


Delhi (India), 1995.

[3]

Bhatia, Rajendra Fourier Series. Hindustan Book Agency, New Delhi


(India), 1993.

[4]

Billingsley, Patrick Probability and Measure. 3rd Edition, John Wiley


and Sons, New York, 1995.

[5]

Friedman, A. Foundations of Modern Analysis. Holt, Rinehart and


Winston, Inc., New York, 1970.

[6]

Halmos, P.R. Measure Theory. Van Nostrand, Princeton, 1950.

[7]

Halmos, P.R. Naive Set Theory. Van Nostrand, Princeton, 1960.

[8]

Hewitt, E. and Stromberg, K. Real and Abstract Analysis. SpringerVerlag, Heidelberg, 1969.

[9]

Kolmogorov, A.N. Foundations of Probability Theory. Chelsea Publishing Company, New York, 1950.

105

106

References

[10] Parthasarathy, K.R. Introduction to Probability and Measure.


Macmillan Company of India Ltd., Delhi, 1977.
[11] Parthasarathy, K. R. Probablity Measures on Metric Spaces, Academic
Press, New York, 1967.
[12] Rana, Inder K. An Introduction to Measure and Integration,
(First Edition) Narosa Publishers, Delhi, 1997.
(Second Edition) Graduate Seies in Mathematics Volume 45, American Mathematical Society, USA, 2001.
[13] Rana, Inder K. From Numbers to Analysis,
World Scientific Publishers, Singapore, 1998.

[14] Royden, H.L. Real Analysis (3rd Edition). Macmillan, New York,
1963.

Index

(X, S, ), 26
C(R), 61
C[a, b], 60
Cc (R), 61
E y , 66
Ex , 66
Lr1 (X, S, ), 80
L1 (X, S, ), 50
L1 [a, b], 56
L1 -metric, 60
L1 (E), 56
Lp (X, S, ), 82
Lp (), 82
S , 89
C, 79
L, 44
L+ , 39
L+
0 , 35
R, 10
I, 2
Im (f ), 79
hf, gi, 88
R
f d, 39, 80
R
sd, 36
R
f d, 40
RE
E f d, 53
, 21
F , 14
Re (f ), 79
- algebra of Borel subsets of R, 27
- finite set function, 17
-algebra generated, 5
-algebra monotone class theorem, 7
-algebra of Borel subsets of X, 7
-algebra technique, 7

-algebra, product, 64
-set, 3
f g, 59
f 1 (C), 3
s1 s2 , 37
s1 s2 , 37
AB, 64
BX , 7
BR , 27
BR2 , 65
C E, 3
F (C), 3
I, 10
I0 , 32
Id , 7
Ir , 7
LF , 24
LR , 27
M(C), 6
S(C), 5
S , 23
pth -norm of f , 82
a.e., 39
a.e. ()x, 39
a.e. ()x Y , 39
a.e. x(), 39
aleph-nought, 103
algebra, 1
algebra generated, 3
almost everywhere, 39
analytic set, 28
arrangement, 99
Arzelas theorem, 58

107

108

Banach spaces, 84
Bessels inequality, 92, 94
Binomial distribution, 10
Borel measurable function, 74
Borel subsets, 27, 33
Borel subsets of R2 , 65
bounded convergence theorem, 52
bounded linear functional, 90
cardinal number, 103
cardinality of a set, 103
cardinality of the continuum, 103
Cartesian product, 101
Cauchy-Schwartz inequality, 83, 88
Chebyshevs inequality, 54, 85
choice function, 102
closed subspace, 89
complete measure space, 26
completion of a measure space, 25, 26
complex numbers, 79
continuity from above, 16
continuity from below, 16
continuum hypothesis, 104
convergent to +, 98
convergent to , 98
countable, 103
countably additive, 9
countably subadditive, 10
countably subadditivity, 12
counting measure, 70
cylinder set, 4
discrete measure, 10
discrete probability measure, 10
distribution function, 15, 32
distribution function, probability, 32
distribution of the measurable function, 48
distribution, Binomial, 10
distribution, discrete probability, 10
distribution, Poisson, 10
distribution, uniform, 10
equipotent, 103
equivalent, 59
essential supremum, 86
essentially bounded, 86
Eulers identity, 96
example, Vitalis, 34
extended real numbers, 10, 98
extension, 19
extension of a measure, 19, 24
Fatous lemma, 45
finite additive property, 11
finite set, 103
finitely additive, 9
Fourier coefficients, 93

Index

Fourier series, 93
Fubinis theorem, 69
function, choice, 102
function, imaginary part of a, 79
function, integrable, 50
function, length, 11
function, measurable, 44
function, nonnegative measurable, 35, 39
function, nonnegative simple measurable, 35
function, real part of a, 79
function, simple measurable, 44
function, support of a, 61
function,integrable, 80
generated, -algebra, 5
generated, algebra, 3
generated, monotone class, 6
graph of the function, 68
H
olders inequality, 83
Haar measure, 31
Hilbert space, 89
inner product, 88
integrable, 50, 80
integral, 39, 50, 80
integral of nonnegative simple measurable
function, 36
integral over E , 40
integration of radial functions, 76
intervals with dyadic endpoints, 7
intervals with rational endpoints, 7
kernel, 91
Lebesgue integrable functions, 56
Lebesgue integral, 56
Lebesgue measurable sets, 27
Lebesgue measurable subsets of R2 , 75
Lebesgue measure, 27
Lebesgue measure space, 27, 75
Lebesgue null, 32
Lebesgue outer measure, 27
Lebesgues dominated convergence, 81
Lebesgue-Stieltjes measure, 32
left open, right closed intervals, 2
length function, 11
length function, countable additivity of, 12
length function, countable subadditivity of,
12
length function, finite additivity of, 11
length function, monotonicity property of,
11
length function, translation invariance of, 12
limit inferior, 98
limit superior, 98

109

Index

mean value property, 57


measurable space, 25
measurable cover, 25
measurable function, 44
measurable function, nonnegative, 39
measurable kernel, 25
measurable partition, 41
measurable rectangle, 64, 65
measurable set, 23
measure, 10
measure space, 25
measure space, complete, 26
measure space, completion of a, 26
measure space,completeness of a, 25
measure, counting, 70
measure, discrete, 10
measure, Haar, 31
measure, Lebesgue, 27
measure, Lebesgue-Stieltjes, 24, 32
measure,outer, 21
measure,outer regular, 33
Minkowskis inequality, 83
monotone class, 5
monotone class generated, 6
monotone convergence theorem, 40
monotonicity property, 11
nonnegative simple measurable function, 35
norm, 59, 84
norm, induced by the inner product, 88
normed linear space, 84
null subset, 26
open intervals, 7, 32
orthogonal, 89
orthogonal complement, 89
outer measure, 21
outer measure induced, 21
outer measure, , 27
outer measure, Lebesgue, 27
outer regular, 33
outer-regularity of , 30
parallelogram identity, 92
Parsevals identity, 95
partial sum of the Fourier series, 93
partition, measurable, 41
Poisson distribution, 10
power set, 1
probability, 26
probability distribution function, 32
probability measure, discrete, 10
probability space, 26
product -algebra, 64, 65
product measure space, 65
product of the measures and , 65
projection theorem, 90

Pythagoras identity, 92
real part, 79
regularity of R2 , 76
representation, 35
Riemann-Lebesgue lemma, 61
Riesz representation, 91
Riesz-Fischer theorem, 83, 95
section of E at x, 66
section of E at y, 66
semi-algebra, 1
set function, 9
set function, -finite, 17
set function, countably additive, 9
set function, countably subadditive, 10
set function, finite, 17
set function, finitely additive, 9
set function, induced, 14
set function, monotone, 9
sigma algebra, 5
simple function technique, 52
simple measurable function, 44
simple,nonnegative measurable function, 35
space, Banach, 84
space, Hilbert, 88
space, inner product, 88
space, Lebesgue measure, 27
space, measurable, 25
space, measure, 25
space, normed, 84
space, probability, 26
space, product measure, 65
standard representation, 35
subset,null, 26
subspace, 89
subspace, closed, 89
supp (f ), 61
support, 61
Theorem, -algebra monotone class, 7
Theorem, Arzelas, 58
theorem, bounded convergence, 52
Theorem, Fubini, 69
Theorem, Lebesgues dominated convergence,
51
Theorem, Monotone Convergence, 40
Theorem, Riesz-Fischer, 60, 95
Theorem, Ulams, 20
topological group, 31
totally finite, 17
transition measure, 75
translation invariance, 12, 75
triangle-inequality, 84
truncation sequence, 48
Ulams theorem, 20

110

ultrafilter, 13
uncountable, 103
Uniform distribution, 10
Vitalis example, 34

Index

Vous aimerez peut-être aussi